Rohen's Photographic Anatomy Flash Cards [2 ed.] 1451194501, 9781451194500

Derived from the leading dissection atlas Anatomy: A Photographic Atlas by Johannes W. Rohen, Chihiro Yokochi, and Elke

992 137 11MB

English Pages 450 Year 2015

Report DMCA / Copyright

DOWNLOAD FILE

Polecaj historie

Rohen's Photographic Anatomy Flash Cards [2 ed.]
 1451194501, 9781451194500

Table of contents :
Rohen's PHOTOGRAPHIC ANATOMY Flash Cards
Title Page
Copyright
Contents
Acknowledgments
Preface
Thorax
1.1
I. LABELS
II. QUESTION
1.2
I. LABELS
II. CLINICAL ANATOMY
1.3
I. LABELS
II. QUESTION
1.4
I. LABELS
II. CLINICAL ANATOMY
1.5
I. LABELS
II. QUESTION
1.6
I. LABELS
II. CLINICAL ANATOMY
1.7
I. LABELS
II. QUESTION
1.8
I. LABELS
II. CLINICAL ANATOMY
1.9
I. LABELS
II. QUESTION
1.10
I. LABELS
II. CLINICAL ANATOMY
1.11
I. LABELS
II. CLINICAL ANATOMY
1.12
I. LABELS
II. QUESTION
1.13
I. LABELS
II. QUESTION
1.14
I. LABELS
II. CLINICAL ANATOMY
1.15
I. LABELS
II. QUESTION
1.16
I. LABELS
II. CLINICAL ANATOMY
1.17
I. LABELS
II. CLINICAL ANATOMY
1.18
I. LABELS
II. QUESTION
1.19
I. LABELS
II. CLINICAL ANATOMY
1.20
I. LABELS
II. CLINICAL ANATOMY
1.21
I. LABELS
II. QUESTION
Abdomen
2.1
I. LABELS
II. QUESTION
2.2
I. LABELS
II. CLINICAL ANATOMY
2.3
I. LABELS
II. QUESTION
2.4
I. LABELS
II. QUESTION
2.5
I. LABELS
II. CLINICAL ANATOMY
2.6
I. LABELS
II. CLINICAL ANATOMY
2.7
I. LABELS
II. CLINICAL ANATOMY
2.8
I. LABELS
II. QUESTION
2.9
I. LABELS
II. CLINICAL ANATOMY
2.10
I. LABELS
II. CLINICAL ANATOMY
2.11
I. LABELS
II. QUESTION
2.12
I. LABELS
II. CLINICAL ANATOMY
2.13
I. LABELS
II. QUESTION
2.14
I. LABELS
II. QUESTION
2.15
I. LABELS
II. CLINICAL ANATOMY
2.16
I. LABELS
II. CLINICAL ANATOMY
2.17
I. LABELS
II. QUESTION
2.18
I. LABELS
II. CLINICAL ANATOMY
2.19
I. LABELS
II. QUESTION
2.20
I. LABELS
II. CLINICAL ANATOMY
2.21
I. LABELS
II. QUESTION
2.22
I. LABELS
II. CLINICAL ANATOMY
2.23
I. LABELS
II. CLINICAL ANATOMY
2.24
I. LABELS
II. QUESTION
2.25
I. LABELS
II. CLINICAL ANATOMY
Pelvis and Perineum
3.1
I. LABELS
II. CLINICAL ANATOMY
3.2
I. LABELS
II. CLINICAL ANATOMY
3.3
I. LABELS
II. QUESTION
3.4
I. LABELS
II. CLINICAL ANATOMY
3.5
I. LABELS
II. QUESTION
3.6
I. LABELS
II. QUESTION
3.7
I. LABELS
II. QUESTION
3.8
I. LABELS
II. CLINICAL ANATOMY
3.9
I. LABELS
II. QUESTION
3.10
I. LABELS
II. CLINICAL ANATOMY
3.11
I. LABELS
II. QUESTION
3.12
I. LABELS
II. CLINICAL ANATOMY
3.13
I. LABELS
II. QUESTION
3.14
I. LABELS
II. QUESTION
3.15
I. LABELS
II. QUESTION
3.16
I. LABELS
II. CLINICAL ANATOMY
Back
4.1
I. LABELS
II. QUESTION
4.2
I. LABELS
II. CLINICAL ANATOMY
4.3
I. LABELS
II. QUESTION
4.4
I. LABELS
II. QUESTION
4.5
I. LABELS
II. QUESTION
4.6
I. LABELS
II. QUESTION
4.7
I. LABELS
II. CLINICAL ANATOMY
4.8
I. LABELS
II. QUESTION
4.9
I. LABELS
II. QUESTION
4.10
I. LABELS
II. CLINICAL ANATOMY
4.11
I. LABELS
II. CLINICAL ANATOMY
4.12
I. LABELS
II. QUESTION
Lower Limb
5.1
I. LABELS
II. CLINICAL ANATOMY
5.2
I. LABELS
II. QUESTION
5.3
I. LABELS
II. QUESTION
5.4
I. LABELS
II. QUESTION
5.5
I. LABELS
II. QUESTION
5.6
I. LABELS
II. CLINICAL ANATOMY
5.7
I. LABELS
II. CLINICAL ANATOMY
5.8
I. LABELS
II. QUESTION
5.9
I. LABELS
II. CLINICAL ANATOMY
5.10
I. LABELS
II. QUESTION
5.11
I. LABELS
II. QUESTION
5.12
I. LABELS
II. CLINICAL ANATOMY
5.13
I. LABELS
II. CLINICAL ANATOMY
5.14
I. LABELS
II. QUESTION
5.15
I. LABELS
II. QUESTION
5.16
I. LABELS
II. CLINICAL ANATOMY
5.17
I. LABELS
II. QUESTION
5.18
I. LABELS
II. QUESTION
5.19
I. LABELS
II. QUESTION
5.20
I. LABELS
II. CLINICAL ANATOMY
5.21
I. LABELS
II. CLINICAL ANATOMY
5.22
I. LABELS
II. QUESTION
5.23
I. LABELS
II. QUESTION
5.24
I. LABELS
II. CLINICAL ANATOMY
5.25
I. LABELS
II. CLINICAL ANATOMY
5.26
I. LABELS
II. CLINICAL ANATOMY
5.27
I. LABELS
II. QUESTION
5.28
I. LABELS
II. CLINICAL ANATOMY
5.29
I. LABELS
II. QUESTION
5.30
I. LABELS
II. CLINICAL ANATOMY
5.31
I. LABELS
II. QUESTION
5.32
I. LABELS
II. QUESTION
5.33
I. LABELS
II. CLINICAL ANATOMY
5.34
I. LABELS
II. CLINICAL ANATOMY
5.35
I. LABELS
II. QUESTION
5.36
I. LABELS
II. CLINICAL ANATOMY
5.37
I. LABELS
II. QUESTION
5.38
I. LABELS
II. QUESTION
5.39
I. LABELS
II. CLINICAL ANATOMY
5.40
I. LABELS
II. QUESTION
5.41
I. LABELS
II. CLINICAL ANATOMY
5.42
I. LABELS
II. QUESTION
Upper Limb
6.1
I. LABELS
II. QUESTION
6.2
I. LABELS
II. CLINICAL ANATOMY
6.3
I. LABELS
II. QUESTION
6.4
I. LABELS
II. QUESTION
6.5
I. LABELS
II. CLINICAL ANATOMY
6.6
I. LABELS
II. QUESTION
6.7
I. LABELS
II. CLINICAL ANATOMY
6.8
I. LABELS
II. QUESTION
6.9
I. LABELS
II. CLINICAL ANATOMY
6.10
I. LABELS
II. QUESTION
6.11
I. LABELS
II. CLINICAL ANATOMY
6.12
I. LABELS
II. QUESTION
6.13
I. LABELS
II. QUESTION
6.14
I. LABELS
II. QUESTION
6.15
I. LABELS
II. CLINICAL ANATOMY
6.16
I. LABELS
II. QUESTION
6.17
I. LABELS
II. QUESTION
6.18
I. LABELS
II. CLINICAL ANATOMY
6.19
I. LABELS
II. QUESTION
6.20
I. LABELS
II. QUESTION
6.21
I. LABELS
II. CLINICAL ANATOMY
6.22
I. LABELS
II. QUESTION
6.23
I. LABELS
II. QUESTION
6.24
I. LABELS
II. QUESTION
6.25
I. LABELS
II. CLINICAL ANATOMY
6.26
I. LABELS
II. QUESTION
6.27
I. LABELS
II. QUESTION
6.28
I. LABELS
II. CLINICAL ANATOMY
6.29
I. LABELS
II. CLINICAL ANATOMY
6.30
I. LABELS
II. QUESTION
6.31
I. LABELS
II. QUESTION
6.32
I. LABELS
II. QUESTION
6.33
I. LABELS
II. CLINICAL ANATOMY
6.34
I. LABELS
II. CLINICAL ANATOMY
6.35
I. LABELS
II. CLINICAL ANATOMY
Head and Neck
7.1
I. LABELS
II. CLINICAL ANATOMY
7.2
I. LABELS
II. QUESTION
7.3
I. LABELS
II. CLINICAL ANATOMY
7.4
I. LABELS
II. QUESTION
7.5
I. LABELS
II. QUESTION
7.6
I. LABELS
II. CLINICAL ANATOMY
7.7
I. LABELS
II. QUESTION
7.8
I. LABELS
II. CLINICAL ANATOMY
7.9
I. LABELS
II. CLINICAL ANATOMY
7.10
I. LABELS
II. QUESTION
7.11
I. LABELS
II. CLINICAL ANATOMY
7.12
I. LABELS
II. QUESTION
7.13
I. LABELS
II. QUESTION
7.14
I. LABELS
II. QUESTION
7.15
I. LABELS
II. CLINICAL ANATOMY
7.16
I. LABELS
II. CLINICAL ANATOMY
7.17
I. LABELS
II. CLINICAL ANATOMY
7.18
I. LABELS
II. CLINICAL ANATOMY
7.19
I. LABELS
II. QUESTION
7.20
I. LABELS
II. CLINICAL ANATOMY
7.21
I. LABELS
II. QUESTION
7.22
I. LABELS
II. CLINICAL ANATOMY
7.23
I. LABELS
II. CLINICAL ANATOMY
7.24
I. LABELS
II. QUESTION
7.25
I. LABELS
II. QUESTION
7.26
I. LABELS
II. QUESTION
7.27
I. LABELS
II. QUESTION
7.28
I. LABELS
II. QUESTION
7.29
I. LABELS
II. QUESTION
7.30
I. LABELS
II. CLINICAL ANATOMY
7.31
I. LABELS
II. CLINICAL ANATOMY
7.32
I. LABELS
II. QUESTION
7.33
I. LABELS
II. QUESTION
7.34
I. LABELS
II. CLINICAL ANATOMY
7.35
I. LABELS
II. CLINICAL ANATOMY
7.36
I. LABELS
II. CLINICAL ANATOMY
7.37
I. LABELS
II. QUESTION
7.38
I. LABELS
II. QUESTION
7.39
I. LABELS
II. CLINICAL ANATOMY
7.40
I. LABELS
II. QUESTION
7.41
I. LABELS
II. QUESTION
7.42
I. LABELS
II. CLINICAL ANATOMY
7.43
I. LABELS
II. CLINICAL ANATOMY
Brain
8.1
I. LABELS
II. CLINICAL ANATOMY
8.2
I. LABELS
II. QUESTION
8.3
I. LABELS
II. QUESTION
8.4
I. LABELS
II. CLINICAL ANATOMY
8.5
I. LABELS
II. CLINICAL ANATOMY
8.6
I. LABELS
II. QUESTION
8.7
I. LABELS
II. QUESTION
8.8
I. LABELS
II. CLINICAL ANATOMY
8.9
I. LABELS
II. QUESTION
8.10
I. LABELS
II. QUESTION
8.11
I. LABELS
II. CLINICAL ANATOMY
8.12
I. LABELS
II. QUESTION
8.13
I. LABELS
II. CLINICAL ANATOMY
8.14
I. LABELS
II. QUESTION
8.15
I. LABELS
II. QUESTION
8.16
I. LABELS
II. CLINICAL ANATOMY
8.17
I. LABELS
II. QUESTION
8.18
I. LABELS
II. CLINICAL ANATOMY
8.19
I. LABELS
II. QUESTION
8.20
I. LABELS
II. QUESTION
8.21
I. LABELS
II. QUESTION
8.22
I. LABELS
II. QUESTION
8.23
I. LABELS
II. CLINICAL ANATOMY
8.24
I. LABELS
II. CLINICAL ANATOMY
8.25
I. LABELS
II. QUESTION
8.26
I. LABELS
II. CLINICAL ANATOMY

Citation preview

Prepare for the dissection lab and operating room with this proven atlas!

Anatomy

A Photographic Atlas 8th Edition Johannes W. Rohen, MD Chihiro Yokochi, PhD Elke Lütjen-Drecoll, MD 978-1-4511-9318-3

Featuring outstanding full-color photographs of actual cadaver dissections, Anatomy: A Photographic Atlas depicts anatomic structures more realistically than traditional atlases. Chapters are organized by region in the order of a typical dissection; each chapter presents topographical anatomical structures in a systemic manner. • Authentic photographic reproduction, as seen in the dissection lab and on the operating table, helps you develop an understanding of the anatomy of the human body. • Functional connections between single organs, the surrounding tissue, and organ systems are clarified to prepare you for lab and practical exams. • Clinical cases and more than 1,200 images including modern MRIs as well as CTs and endoscopic techniques. • Dissections illustrate the topographical anatomy in layers “from the outside in.”

Vilensky_FM.indd i

3/17/15 12:35 AM

Anatomy in real-life 3D! Acland’s Video Atlas of Human Anatomy Dr. Robert D. Acland 978-1451145007 (12-month access card)

AclandAnatomy.com is Lippincott’s online portal to more than 300 video clips showing real human anatomic specimens in their natural colors. Clear narration, labeled structures, and Q&A reinforce learning and review. Unique for its rotating videography and carefully dissected specimens, the site allows students to see anatomy in context and prepares them for the lab and the practical exam. • Browse the videos by region or topic; the site navigation makes it easy to find the content you need to prepare for class and lab or to review for exams. • Download the PDF transcripts for offline review, or save your favorite clips for reference later. • Take a quiz in review or exam/timed mode and track your performance; quiz results include a link to the supporting video clip. • View subscription options and free clips at AclandAnatomy.com, including groundbreaking new footage of the cochlea (inner ear)!

Vilensky_FM.indd ii

3/17/15 12:35 AM

Rohen’s

PHOTOGRAPHIC ANATOMY Flash Cards Joel A. Vilensky, PhD Professor, Department of Anatomy and Cell Biology Indiana University School of Medicine Fort Wayne, Indiana

Leslie A. Hoffman, PhD Assistant Professor, Department of Anatomy and Cell Biology Indiana University School of Medicine Fort Wayne, Indiana

Vilensky_FM.indd iii

3/17/15 12:35 AM

Acquisitions Editor: Crystal Taylor Product Development Editor: Greg Nicholl Marketing Manager: Joy Fisher Williams Production Project Manager: Priscilla Crater Design Coordinator: Teresa Mallon Art Director: Jennifer Clements Prepress Vendor: Absolute Service, Inc. 2nd edition Copyright © 2016 Wolters Kluwer Copyright © 2009 Lippincott Williams & Wilkins, a Wolters Kluwer business. All rights reserved. This book is protected by copyright. No part of this book may be reproduced or transmitted in any form or by any means, including as photocopies or scanned-in or other electronic copies, or utilized by any information storage and retrieval system without written permission from the copyright owner, except for brief quotations embodied in critical articles and reviews. Materials appearing in this book prepared by individuals as part of their official duties as U.S. government employees are not covered by the above-mentioned copyright. To request permission, please contact Wolters Kluwer at Two Commerce Square, 2001 Market Street, Philadelphia, PA 19103, via email at [email protected], or via our website at lww.com (products and services). 987654321 Printed in China ISBN: 978-1-4511-9450-0 This work is provided “as is,” and the publisher disclaims any and all warranties, express or implied, including any warranties as to accuracy, comprehensiveness, or currency of the content of this work. This work is no substitute for individual patient assessment based upon healthcare professionals’ examination of each patient and consideration of, among other things, age, weight, gender, current or prior medical conditions, medication history, laboratory data and other factors unique to the patient. The publisher does not provide medical advice or guidance and this work is merely a reference tool. Healthcare professionals, and not the publisher, are solely responsible for the use of this work including all medical judgments and for any resulting diagnosis and treatments. Given continuous, rapid advances in medical science and health information, independent professional verification of medical diagnoses, indications, appropriate pharmaceutical selections and dosages, and treatment options should be made and healthcare professionals should consult a variety of sources. When prescribing medication, healthcare professionals are advised to consult the product information sheet (the manufacturer’s package insert) accompanying each drug to verify, among other things, conditions of use, warnings and side effects and identify any changes in dosage schedule or contraindications, particularly if the medication to be administered is new, infrequently used or has a narrow therapeutic range. To the maximum extent permitted under applicable law, no responsibility is assumed by the publisher for any injury and/or damage to persons or property, as a matter of products liability, negligence law or otherwise, or from any reference to or use by any person of this work. LWW.com

Vilensky_FM.indd iv

3/17/15 12:35 AM

Contents Acknowledgments Preface

Vilensky_FM.indd v

1

THORAX

2

ABDOMEN

3

PELVIS AND PERINEUM

4

BACK

5

LOWER LIMB

6

UPPER LIMB

7

HEAD AND NECK

8

BRAIN

3/17/15 12:35 AM

Acknowledgments We wish to thank Ms. Crystal Taylor and Mr. Greg Nicholl at Wolters Kluwer for inviting us to compose these cards and assisting us with the process. Ms. Haley Moon and Ms. Lowene Stipp are thanked for their assistance with some of the logistical issues involved in assembling these cards. And we are grateful to our students for teaching us how to best use these cards to help them learn anatomy.

Vilensky_FM.indd vii

3/17/15 12:35 AM

Preface All of the photographs in this flash card set were derived from the 8th edition of Anatomy: A Photographic Atlas by Johannes W. Rohen, Chihiro Yokochi, and Elke Lütjen-Drecoll (2016). We chose a subset of the large number of images in that atlas to use in this flash card set. This subset primarily includes images that seemed to us to be of most relevance to students preparing for a practical exam in gross anatomy and neuroanatomy. Furthermore, we greatly reduced the number of structures labeled in the flash cards compared to those in the atlas (although increased in this edition compared to the number of structures labeled in the first edition of the cards). Also in this edition, we added material that will be useful to the students in their preparation for the “written” component of their gross anatomy exams. The back of each card, in addition to having the answers to the structures labeled with numbers on the front, includes either an anatomical question involving one or more of the labeled structures or a related clinical anatomy “pearl” that is likely to be tested on their exam. We recognize that the images on these cards lack the three-dimensional definition that cadavers offer; however, the dissections used for these photographs are so good that this liability is not as significant as it might be with lesser quality images.

Vilensky_FM.indd ix

3/17/15 12:35 AM

Performing well on a practical exam is a skill that some students find difficult to master, and we hope that students find these cards useful for acquiring this skill. Joel A. Vilensky, PhD Leslie A. Hoffman, PhD

Vilensky_FM.indd x

3/17/15 12:35 AM

Thorax

1.1

1 4

2

5

6 3

Vilensky_01.indd 1

3/17/15 12:47 AM

1.1

I. LABELS 1. 2. 3. 4. 5. 6.

Deep fascia overlying pectoralis major Mammary gland Serratus anterior Lactiferous sinus Nipple Areola

II. QUESTION In your patient with breast cancer, you are concerned about possible spread of cancer cells (metastasis) via the lymphatic system. Most of the lymph from the breast first travels to which of the following groups of lymph nodes? Parasternal nodes Pectoral (anterior) group of axillary nodes Subscapular group of axillary nodes Phrenic nodes Tracheobronchial nodes

Copyright © 2016 Wolters Kluwer

Vilensky_01.indd 2

Answer is B

A. B. C. D. E.

3/17/15 12:47 AM

Thorax

1.2 5 6

1 2

3

7 8

4

Vilensky_01.indd 3

3/17/15 12:47 AM

I. LABELS 1. 2. 3. 4. 5. 6. 7. 8.

1.2

Internal intercostal Intercostal artery and vein Rectus abdominis External abdominal oblique Cephalic vein Pectoralis major Linea alba Anterior layer of rectus sheath

II. CLINICAL ANATOMY Movements of the thoracic wall during inspiration increase the volume of the thoracic cavity, resulting in a pressure gradient that draws air into the lungs. The volume of the thoracic cavity increases in three dimensions: vertically, as a result of contraction of the diaphragm; transversely, as a result of contraction of intercostal muscles (bucket-handle movement); and anteroposteriorly, as a result of elevation of the anterior ends of the ribs and the sternum (pump-handle movement).

Copyright © 2016 Wolters Kluwer

Vilensky_01.indd 4

3/17/15 12:47 AM

Thorax

1.3

1

2 5 3

4

Vilensky_01.indd 5

6

3/17/15 12:47 AM

1.3

I. LABELS 1. 2. 3. 4. 5. 6.

Clavicular head of pectoralis major Sternocostal head of pectoralis major Mammary gland External abdominal oblique Internal thoracic (mammary) artery and vein Superior epigastric artery and vein

II. QUESTION A 78-year-old patient is about to have a coronary bypass operation. The surgeon has chosen to use the patient’s right internal thoracic artery for the operation, which will require cutting the artery. Accordingly, in which of the following arteries would you expect to see a loss of blood pressure when the artery is cut? Vertebral artery Musculophrenic artery Inferior phrenic artery Brachiocephalic trunk Right bronchial artery

Copyright © 2016 Wolters Kluwer

Vilensky_01.indd 6

Answer is B

A. B. C. D. E.

3/17/15 12:47 AM

Thorax

1.4

1

2

4 5

3

6

7

Vilensky_01.indd 7

3/17/15 12:47 AM

I. LABELS 1. 2. 3. 4. 5. 6. 7.

1.4

Clavicle Internal intercostal muscle Transversus thoracis muscle Internal thoracic (mammary) artery and vein Sternum Intercostal artery and vein Diaphragm (divided)

II. CLINICAL ANATOMY The internal thoracic (mammary) artery can be used for a coronary artery bypass procedure. One technique is to free the distal portion of the artery and anastomose it to a coronary artery distal to the blockage.

Copyright © 2016 Wolters Kluwer

Vilensky_01.indd 8

3/17/15 12:47 AM

Thorax

1.5 5

1

2

6

3

7 8

4

Vilensky_01.indd 9

3/17/15 12:47 AM

I. LABELS 1. 2. 3. 4. 5. 6. 7. 8.

1.5

Thymus; atrophic Upper lobe of right lung Middle lobe of right lung Diaphragm Left lobe of thyroid gland Upper lobe of left lung Cardiac notch of left lung Lingula of left lung

II. QUESTION A 68-year-old woman has been diagnosed with lung cancer located near the cardiac notch. Which of the following lung lobes is most likely to be removed by her surgeon? Superior lobe of the right lung Superior lobe of the left lung Inferior lobe of the right lung Middle lobe of the right lung Inferior lobe of the left lung

Copyright © 2016 Wolters Kluwer

Vilensky_01.indd 10

Answer is B

A. B. C. D. E.

3/17/15 12:47 AM

Thorax

1.6

1 5 2

6 7

3 8

4

Vilensky_01.indd 11

3/17/15 12:47 AM

I. LABELS 1. 2. 3. 4. 5. 6. 7. 8.

1.6

Brachiocephalic trunk Right phrenic nerve Probe in the transverse pericardial sinus Diaphragm covering liver Left brachiocephalic vein Superior vena cava Ascending aorta Left phrenic nerve

II. CLINICAL ANATOMY The branches of the aortic arch are variable. In one variant, the right subclavian artery, which normally branches from the brachiocephalic trunk, branches from the descending part of the arch and passes posterior to the esophagus, potentially compressing it and causing difficulty swallowing (dysphagia).

Copyright © 2016 Wolters Kluwer

Vilensky_01.indd 12

3/17/15 12:47 AM

Thorax

1.7

1 7 2 8 3 4

9

5 10 6

11

Vilensky_01.indd 13

3/17/15 12:47 AM

I. LABELS 1. 2. 3. 4. 5. 6. 7. 8. 9. 10. 11.

1.7

Apex of lung Upper lobe of right lung Middle lobe of right lung Lower lobe of right lung Groove of subclavian artery Groove of azygos arch Upper lobe of left lung Oblique fissure of left lung Lower lobe of left lung Groove of aortic arch Cardiac impression

II. QUESTION An 85-year-old man with dyspnea (difficulty breathing) who smoked cigarettes for more than 20 years was diagnosed with a large carcinoma in his left lung. Which of the following structures or features is characteristic of the cancerous lung? Groove for superior vena cava (SVC) Horizontal fissure Three lobes Embedded thoracic duct Lingula

Copyright © 2016 Wolters Kluwer

Vilensky_01.indd 14

B Answer is E

A. B. C. D. E.

3/17/15 12:47 AM

Thorax

1.8

5 1 2 6

3 4

Vilensky_01.indd 15

3/17/15 12:47 AM

I. LABELS 1. 2. 3. 4. 5. 6.

1.8

Cardiac nerves Arch of azygos vein Right vagus nerve on esophagus Inferior vena cava Left vagus nerve Left recurrent laryngeal nerve originating from left vagus nerve

II. CLINICAL ANATOMY The cardiac plexuses supply autonomic innervation to the heart via cardiac nerves from the vagus and recurrent laryngeal nerves (parasympathetic) and the cervical and upper thoracic sympathetic ganglia (sympathetic). The sympathetic fibers cause an increase in heart rate and force of contraction, whereas the parasympathetic fibers act to slow heart rate.

Copyright © 2016 Wolters Kluwer

Vilensky_01.indd 16

3/17/15 12:47 AM

Thorax

1.9

1 4 5 2 3

Vilensky_01.indd 17

6

7

3/17/15 12:47 AM

I. LABELS 1. 2. 3. 4. 5. 6. 7.

1.9

Right brachiocephalic vein (cut) Arch of azygos vein Right pulmonary vein Left common carotid artery Aortic arch Left recurrent laryngeal nerve Left pulmonary artery

II. QUESTION A 76-year-old woman complains to her physician of hoarseness. A CT scan reveals a mass inferior to the arch of her aorta. You suspect her hoarseness is being caused by pressure on which of the following structures? Carina Azygos vein Left common carotid artery Left sympathetic trunk Left recurrent laryngeal nerve

Copyright © 2016 Wolters Kluwer

Vilensky_01.indd 18

B Answer is E

A. B. C. D. E.

3/17/15 12:47 AM

Thorax

1.10

1 5 2

6 3 4

Vilensky_01.indd 19

7

3/17/15 12:47 AM

I. LABELS 1. 2. 3. 4. 5. 6. 7.

1.10

Brachiocephalic trunk Right brachiocephalic vein Right atrium Valve cusp of right atrioventricular valve Left common carotid artery Pulmonary trunk (fenestrated) Anterior papillary muscle of the right ventricle

II. CLINICAL ANATOMY The rough pectinate muscles of the atria are in the auricles. The remainder of the atria has smooth walls. The sinoatrial (SA) node is located in the crista terminalis, which separates the smooth and rough walls. This node is the pacemaker of the heart and receives blood supply usually via an atrial branch of the right coronary artery.

Copyright © 2016 Wolters Kluwer

Vilensky_01.indd 20

3/17/15 12:47 AM

Thorax

1.11

1 2 5 3 4

Vilensky_01.indd 21

6

7

3/17/15 12:47 AM

I. LABELS 1. 2. 3. 4. 5. 6. 7.

1.11

Right posterior intercostal artery Azygos vein Right posterior intercostal vein Inferior vena cava (opening) Bronchial artery Thoracic aorta Esophagus

II. CLINICAL ANATOMY The thoracic duct is a large lymphatic vessel that traverses the posterior mediastinum between the azygos vein and the aorta, posterior to the esophagus. Injury to the thoracic duct causes chylothorax (accumulation of chyle in the thoracic and pleural cavities).

Copyright © 2016 Wolters Kluwer

Vilensky_01.indd 22

3/17/15 12:47 AM

Thorax

1

1.12

6

2 3 4

7

5

Vilensky_01.indd 23

3/17/15 12:47 AM

1.12

I. LABELS 1. 2. 3. 4. 5. 6. 7.

Superior vena cava Right atrium Right ventricle Costal part of diaphragm Lumbar part of diaphragm Ascending aorta Left ventricle

II. QUESTION A 14-year-old boy was admitted to a local hospital with a previous history of heart failure. His enlarged and thin-walled left ventricle is most likely associated with which of the following conditions? A stenotic pulmonary trunk A stenotic ascending aorta A constricted left atrioventricular aperture A constricted right atrioventricular aperture Inadequate closure of the mitral valve

Copyright © 2016 Wolters Kluwer

Vilensky_01.indd 24

Answer is B

A. B. C. D. E.

3/17/15 12:47 AM

Thorax

1.13

1

2 5 3

6 4

7

Vilensky_01.indd 25

3/17/15 12:47 AM

I. LABELS 1. 2. 3. 4. 5. 6. 7.

1.13

Brachiocephalic trunk Pulmonary trunk Right auricle Right ventricle Left auricle Left ventricle Apex of heart

II. QUESTION Transposition of the great vessels is a congenital heart defect in which the locations of the aorta and pulmonary trunk are reversed, decreasing the amount of oxygen supplied to the body. This condition can be compatible with life for a short time if it is accompanied by which of the following defects? Aortic atresia Coarctation of the aorta Persistent umbilical vein Patent ductus arteriosus Tetralogy of Fallot

Copyright © 2016 Wolters Kluwer

Vilensky_01.indd 26

B Answer is D

A. B. C. D. E.

3/17/15 12:47 AM

Thorax

1.14 4

1

5 2 3

Vilensky_01.indd 27

6

3/17/15 12:47 AM

I. LABELS

1.14

Ascending aorta Right atrium Right coronary artery traversing coronary groove Left common carotid artery and left subclavian artery 5. Pulmonary trunk 6. Anterior interventricular branch of left coronary artery

1. 2. 3. 4.

II. CLINICAL ANATOMY Three cardiac veins usually drain into the coronary sinus, which in turn drains into the right atrium: great, middle, and small cardiac veins. In addition, the anterior cardiac veins drain directly into the right atrium, and the Thebesian veins (venae cordis minimae) are very small veins that drain into all four chambers. The larger veins can be used to guide the electrodes of an implantable pacemaker.

Copyright © 2016 Wolters Kluwer

Vilensky_01.indd 28

3/17/15 12:47 AM

Thorax

1.15

1

2

4 5

3 6

Vilensky_01.indd 29

3/17/15 12:47 AM

I. LABELS 1. 2. 3. 4. 5. 6.

1.15

Pulmonic valve Left coronary artery Left atrioventricular (mitral) valve Aortic valve Right coronary artery Right atrioventricular (tricuspid) valve

II. QUESTION A 42-year-old woman with an irregular heartbeat sees her doctor for her annual checkup. Where should her physician place the stethoscope to hear the opening and closing of the mitral valve? A. In the left fifth intercostal space at the midclavicular line B. In the left fourth intercostal space at the midclavicular line C. In the second right intercostal space just adjacent to the sternum D. In the second left intercostal space just adjacent to the sternum E. In the eighth left intercostal space immediately adjacent to the xiphoid process

Vilensky_01.indd 30

B Answer is A

Copyright © 2016 Wolters Kluwer

3/17/15 12:47 AM

Thorax

1.16

1

2

4

3

5 6

Vilensky_01.indd 31

3/17/15 12:47 AM

I. LABELS

1.16

Aorta Right coronary artery Right marginal branch of right coronary artery Anterior interventricular branch of left coronary artery 5. Right coronary artery 6. Posterior interventricular branch of right coronary artery

1. 2. 3. 4.

II. CLINICAL ANATOMY Coronary dominance is determined by the coronary artery that gives rise to the posterior interventricular artery. The right coronary artery is dominant in approximately 70% of the population, whereas the left coronary artery is dominant in about 10% of the population. Approximately 20% of hearts exhibit codominance, meaning that both the right coronary artery and the circumflex branch of the left coronary artery supply the posterior interventricular artery.

Copyright © 2016 Wolters Kluwer

Vilensky_01.indd 32

3/17/15 12:47 AM

Thorax 1

1.17 2

3

Vilensky_01.indd 33

3/17/15 12:47 AM

I. LABELS

1.17

1. Ascending aorta 2. Pulmonary trunk 3. Descending aorta

II. CLINICAL ANATOMY In axial radiological cross-sectional images, a unitary structure may appear to be more than a single structure if it curves in and out of the plane of section. So in this axial image, we see the ascending and descending aorta but do not see the higher section of the aortic arch that unites the two.

Copyright © 2016 Wolters Kluwer

Vilensky_01.indd 34

3/17/15 12:47 AM

Thorax

1.18 2 3

1 4 5

Vilensky_01.indd 35

3/17/15 12:47 AM

I. LABELS 1. 2. 3. 4. 5.

1.18

Right atrium Right ventricle Interventricular septum Left atrium Descending aorta

II. QUESTION During a routine checkup, a physician discovers a heart murmur in his 53-year-old male patient. He refers the patient to a cardiologist who decides to perform a transesophageal echocardiogram (TEE) to examine the man’s heart valves. Which of the following structures is located closest to the esophagus? Apex of the heart Ascending aorta Pulmonic valve Left atrium Right auricle

Copyright © 2016 Wolters Kluwer

Vilensky_01.indd 36

B Answer is D

A. B. C. D. E.

3/17/15 12:47 AM

Thorax 1

1.19 4 5

2 6

3

Vilensky_01.indd 37

7

3/17/15 12:47 AM

I. LABELS 1. 2. 3. 4. 5. 6. 7.

1.19

Brachiocephalic trunk Superior vena cava Left atrium Left common carotid artery Left subclavian artery Aortic arch Pulmonary veins

II. CLINICAL ANATOMY Coarctation is a narrowing of the aorta, typically occurring at the junction of the aortic arch and the descending aorta. This constriction forces the heart to pump harder than normal in order to propel blood through the stenotic part of the aorta. This results in high blood pressure in the head, neck, and upper limbs and low blood pressure in the lower limbs. Some consequences of coarctation are premature heart disease, cerebral aneurysm, organ failure, arterial rupture, and death.

Copyright © 2016 Wolters Kluwer

Vilensky_01.indd 38

3/17/15 12:48 AM

Thorax

1 2

1.20

5

3 6

7

4

Vilensky_01.indd 39

3/17/15 12:48 AM

I. LABELS

1.20

Ganglion of sympathetic trunk Sympathetic trunk Posterior intercostal vessels and intercostal nerve Thoracic splanchnic nerve contributing to greater splanchnic nerve 5. Right primary bronchus 6. Right pulmonary artery 7. Right pulmonary vein

1. 2. 3. 4.

II. CLINICAL ANATOMY The greater, lesser, and least splanchnic nerves arise from the thoracic sympathetic trunk and enter the abdomen to supply all of the abdominal viscera with sympathetic innervation. Parasympathetic innervation to the foregut and midgut is supplied via the vagus nerve, and the hindgut receives parasympathetic innervation from the pelvic splanchnic nerves. Occasionally, ganglion cells regulating peristalsis do not migrate from the neural crest to the distal colon. This creates a functional blockage of the colon. This congenital condition is called aganglionic megacolon (Hirschsprung’s disease).

Copyright © 2016 Wolters Kluwer

Vilensky_01.indd 40

3/17/15 12:48 AM

Thorax

1.21

5 1 2 3 4

6 7 8

Vilensky_01.indd 41

3/17/15 12:48 AM

I. LABELS 1. 2. 3. 4. 5. 6. 7. 8.

1.21

Left pulmonary artery Left primary bronchus Left pulmonary vein Left phrenic nerve and pericardiacophrenic artery and vein Sympathetic trunk White ramus communicans of sympathetic trunk Thoracic aorta Posterior intercostal artery and vein and intercostal nerve

II. QUESTION A 48-year-old woman presents to the emergency department complaining of severe shortness of breath. Radiographic examination reveals a mass in the apex of her right lung. Physical exam reveals ptosis and miosis in her right eye, along with anhidrosis and flushing on the right side of her face. Which of the following structures is likely being compressed by the mass? Brachiocephalic trunk Right brachiocephalic vein Phrenic nerve Sympathetic trunk Vagus nerve

Copyright © 2016 Wolters Kluwer

Vilensky_01.indd 42

B Answer is D

A. B. C. D. E.

3/17/15 12:48 AM

Abdomen

2.1

1 4

5 2

3 6

Vilensky_02.indd 43

3/17/15 12:22 AM

I. LABELS 1. 2. 3. 4. 5. 6.

2.1

Rectus abdominis muscle Inferior epigastric artery and vein Superficial inguinal lymph nodes Linea alba Internal abdominal oblique muscle Spermatic cord

II. QUESTION A young male patient presents with a lower abdominal mass that you diagnose as an indirect inguinal hernia. Which of the following is correct about such hernias? A. The sac of the hernia passes lateral to the inferior epigastric artery. B. The sac of the hernia passes medial to the inferior epigastric artery. C. The hernial sac is typically embedded in the superficial inguinal lymph nodes. D. The hernia exits the abdomen through the linea alba. E. The hernia exits the abdomen through the fibers of the rectus abdominis.

Vilensky_02.indd 44

Answer is A

Copyright © 2016 Wolters Kluwer

3/17/15 12:22 AM

Abdomen

2.2

1

2 3

4 5

6

Vilensky_02.indd 45

3/17/15 12:22 AM

I. LABELS 1. 2. 3. 4. 5. 6.

2.2

Thoracoabdominal (intercostal) nerve Posterior layer of rectus sheath Transverse abdominal muscle Arcuate line Inferior epigastric artery Inguinal ligament

II. CLINICAL ANATOMY The composition of the rectus sheath changes at the arcuate line. Inferior to this line, the posterior lamina of the sheath is composed only of transversalis fascia. Superior to this line, the posterior lamina is composed of part of the aponeurosis of the internal oblique muscle and the aponeurosis of the transverse abdominal muscle.

Copyright © 2016 Wolters Kluwer

Vilensky_02.indd 46

3/17/15 12:22 AM

Abdomen

1

2.3

3 4

2

Vilensky_02.indd 47

3/17/15 12:22 AM

2.3

I. LABELS 1. 2. 3. 4.

Inguinal ligament Ilioinguinal nerve Superficial inguinal ring Round ligament of the uterus

II. QUESTION While performing a hernia repair, a surgeon does not want to destroy the integrity of the superficial inguinal ring. She knows that this ring is an aperture in the: Internal oblique aponeurosis Transverse abdominal muscle aponeurosis External oblique aponeurosis Transversalis fascia Peritoneum

Copyright © 2016 Wolters Kluwer

Vilensky_02.indd 48

Answer is C B

A. B. C. D. E.

3/17/15 12:22 AM

Abdomen

2.4

5 1 2

3 4

Vilensky_02.indd 49

3/17/15 12:22 AM

I. LABELS 1. 2. 3. 4. 5.

2.4

Ductus deferens (spermatic cord) Superficial circumflex iliac artery Femoral vein Superficial inguinal lymph nodes Inferior epigastric artery and vein

II. QUESTION In performing surgery to reduce an aneurysm in the external iliac artery, the surgeon must be careful to avoid damage to the branches of this artery. One such branch is the: External pudendal artery Obturator artery Superior gluteal artery Inferior gluteal artery Inferior epigastric artery

Copyright © 2016 Wolters Kluwer

Vilensky_02.indd 50

Answer is E

A. B. C. D. E.

3/17/15 12:22 AM

Abdomen

2.5

1

2

3

4 5

Vilensky_02.indd 51

3/17/15 12:22 AM

I. LABELS 1. 2. 3. 4. 5.

2.5

Liver Stomach Transverse colon Small intestine Cecum

II. CLINICAL ANATOMY The small intestine is composed of the duodenum, jejunum, and the ileum. Although the latter two are not delineated by a specific point, they are distinguished as follows: • • •

The jejunum has less fat inside its mesentery than the ileum. The jejunum is typically larger in diameter than the ileum. The jejunum has simple arcades and long vasa recta, whereas the ileum has more complex arcades and shorter vasa recta.

Copyright © 2016 Wolters Kluwer

Vilensky_02.indd 52

3/17/15 12:22 AM

Abdomen

2.6

5 1 2 3

6 7

4

Vilensky_02.indd 53

3/17/15 12:22 AM

I. LABELS

2.6

1. Ligamentum teres hepatis (round ligament of liver; reflected) 2. Cystic artery 3. Right gastric artery 4. Right gastro-omental (gastroepiploic) artery 5. Abdominal part of esophagus (cardiac part of stomach) 6. Left gastric artery; branch of celiac trunk 7. Gastroduodenal artery; splits into superior pancreatoduodenal and right gastro-omental arteries

II. CLINICAL ANATOMY In patients with portal hypertension, veins in the ligamentum teres hepatis enlarge and so do their anastomoses with veins in the abdominal wall, producing varicosities in the abdominal wall around the umbilicus. These radiating varices are known as caput medusa.

Copyright © 2016 Wolters Kluwer

Vilensky_02.indd 54

3/17/15 12:22 AM

Abdomen

2.7 5

1 2 6 7

3 4

Vilensky_02.indd 55

3/17/15 12:22 AM

I. LABELS

2.7

1. Gallbladder (fundus) 2. Hepatoduodenal ligament 3. Probe within the epiploic (omental) foramen (of Winslow) 4. Descending part of duodenum 5. Caudate lobe of liver 6. Greater curvature of stomach 7. Lesser omentum

II. CLINICAL ANATOMY The duodenum has four parts: superior, descending, inferior, and ascending (sometimes designated at first, second, third, and fourth). The first three parts are associated with the corresponding lumbar vertebrae (L1, L2, and L3). The distal three parts of the duodenum are retroperitoneal.

Copyright © 2016 Wolters Kluwer

Vilensky_02.indd 56

3/17/15 12:22 AM

Abdomen

2.8

1 2

4

3 5

Vilensky_02.indd 57

3/17/15 12:22 AM

I. LABELS 1. 2. 3. 4. 5.

2.8

Gallbladder Probe within the epiploic (omental) foramen Right colic (hepatic) flexure of the large intestine Head of the pancreas Branches of gastro-omental artery

II. QUESTION Upon putting his finger through the epiploic foramen, the surgeon knows that directly posterior to his finger is: Inferior vena cava (IVC) Common hepatic artery Cystic duct Portal vein Gastroduodenal artery

Copyright © 2016 Wolters Kluwer

Vilensky_02.indd 58

Answer is A

A. B. C. D. E.

3/17/15 12:22 AM

Abdomen

2.9

1 3

2

4 5 6

Vilensky_02.indd 59

3/17/15 12:22 AM

I. LABELS

2.9

1. Fundus of gallbladder 2. Cystic duct 3. Ligamentum teres hepatis (round ligament of the liver) 4. Common hepatic duct 5. Hepatic artery proper 6. Ligamentum venosum

II. CLINICAL ANATOMY Blockage of the biliary tree by a gallstone is typically associated with jaundice, although if just the cystic duct is blocked, jaundice is less likely.

Copyright © 2016 Wolters Kluwer

Vilensky_02.indd 60

3/17/15 12:22 AM

Abdomen 1

2.10 3

4 2

Vilensky_02.indd 61

3/17/15 12:22 AM

I. LABELS 1. 2. 3. 4.

2.10

Cystic duct Common bile duct Pancreatic duct Major duodenal papilla

II. CLINICAL ANATOMY If a gallstone lodges at the apex of the major duodenal papilla (where sphincter of Oddi is located), it will block both the main pancreatic duct and the common bile duct. Subsequently, bile may back up into the pancreas, producing pancreatitis. Pancreatic pain is usually localized in the upper middle or upper left part of the abdomen. It may also radiate to the back.

Copyright © 2016 Wolters Kluwer

Vilensky_02.indd 62

3/17/15 12:22 AM

Abdomen 1

2.11

5

2

3

6

7

4

Vilensky_02.indd 63

3/17/15 12:22 AM

I. LABELS 1. 2. 3. 4. 5. 6. 7.

2.11

Liver Inferior vena cava (IVC) Thoracic aorta Kidney Stomach Spleen Left renal vein

II. QUESTION In the emergency department, an adult patient is shown to have a ruptured spleen after an automobile crash. Which of the following is most likely in this situation? The spleen will be repaired. The splenic artery will be ligated. The celiac trunk will be ligated. A splenectomy will be performed. The spleen will be transplanted.

Copyright © 2016 Wolters Kluwer

Vilensky_02.indd 64

Answer is D B

A. B. C. D. E.

3/17/15 12:22 AM

Abdomen

2.12

1 2 4 5 6 7 3

Vilensky_02.indd 65

3/17/15 12:22 AM

I. LABELS 1. 2. 3. 4. 5. 6. 7.

2.12

Portal vein Common hepatic duct (dilated) Right gastro-omental (gastroepiploic) artery Left kidney Celiac trunk Splenic artery Body of pancreas

II. CLINICAL ANATOMY The anterior boundary of the epiploic (omental) foramen is the hepatoduodenal ligament. Within this ligament are the proper hepatic artery, common bile duct, and portal vein. The artery is to the left, the duct is to the right, and the vein is posterior.

Copyright © 2016 Wolters Kluwer

Vilensky_02.indd 66

3/17/15 12:22 AM

Abdomen

2.13

1

2 3

5

4

Vilensky_02.indd 67

6

3/17/15 12:22 AM

I. LABELS

2.13

1. Ligamentum teres hepatis 2. Cystic artery 3. Right gastric artery; here going to pylorus (cut and reflected) 4. Superior mesenteric artery 5. Splenic vein 6. Main pancreatic duct

II. QUESTION A patient is having severe abdominal pain that is attributable to ischemia. A CT arteriogram reveals that all the branches of the superior mesenteric artery are patent. Therefore, the pain may be a consequence of ischemia in which of the following structures? Ascending colon Transverse colon Descending colon Jejunum Cecum

Copyright © 2016 Wolters Kluwer

Vilensky_02.indd 68

Answer is C

A. B. C. D. E.

3/17/15 12:22 AM

Abdomen

2.14

4 1 2

5

3

6 7

Vilensky_02.indd 69

3/17/15 12:22 AM

I. LABELS 1. 2. 3. 4. 5. 6. 7.

2.14

Proper hepatic artery Cystic duct Gallbladder Splenic artery Tail of pancreas Superior mesenteric artery Ureter

II. QUESTION Following blunt trauma to the abdomen, an emergency laparotomy is performed. Copious bleeding is noted from a lacerated liver. The surgeon inserts an index finger into the epiploic (omental) foramen (of Winslow) and compresses the hepatoduodenal ligament with the thumb (Pringle maneuver). Blood supply to which of the following structures would not be affected by this procedure? Gallbladder Quadrate lobe of liver Caudate lobe of liver Left lobe of liver Pancreas

Copyright © 2016 Wolters Kluwer

Vilensky_02.indd 70

Answer is E

A. B. C. D. E.

3/17/15 12:22 AM

Abdomen

2.15

6 1 2 3 4 5

7

8 9

Vilensky_02.indd 71

3/17/15 12:22 AM

I. LABELS 1. 2. 3. 4. 5. 6. 7. 8. 9.

2.15

Proper hepatic artery Common hepatic artery Gastroduodenal artery Duodenum Gallbladder Celiac trunk Splenic artery Superior mesenteric artery Superior mesenteric vein

II. CLINICAL ANATOMY The splenic artery has a very meandering course along the superior edge of the pancreas, with the vein inferior to it. Because of this meandering course, discontinuous portions of it may be present on single axial CT scans of the upper abdomen. This can be confusing to students who expect to see a continuous structure.

Copyright © 2016 Wolters Kluwer

Vilensky_02.indd 72

3/17/15 12:22 AM

Abdomen

2.16

1 2 3

4 5

6

Vilensky_02.indd 73

3/17/15 12:22 AM

I. LABELS 1. 2. 3. 4. 5. 6.

2.16

Middle colic artery Right colic artery Duodenum Duodenojejunal flexure Superior mesenteric vein Ileal artery

II. CLINICAL ANATOMY The superior mesenteric vein joins with the splenic vein to form the portal vein posterior to the neck of the pancreas. The inferior mesenteric vein is usually a tributary of the superior mesenteric vein and thus does not usually contribute to the formation of the portal vein.

Copyright © 2016 Wolters Kluwer

Vilensky_02.indd 74

3/17/15 12:22 AM

Abdomen

2.17

1

4 5

2 3

6 7 8

Vilensky_02.indd 75

3/17/15 12:22 AM

I. LABELS 1. 2. 3. 4. 5. 6. 7. 8.

2.17

Central tendon of diaphragm Ureter Right gonadal vein Splenic artery Suprarenal (adrenal) gland Left suprarenal vein Left gonadal vein Superior hypogastric plexus; composed mainly of lumbar splanchnic nerves (sympathetic)

II. QUESTION Kidney transplant surgeons prefer to transplant the left kidney compared to the right kidney. Which of the following is true and most likely explains this preference? A. The left renal vein is longer than the right renal vein. B. The left renal artery is longer than the right renal artery. C. The right renal vein receives the right gonadal vein. D. The right renal artery is the only source of blood to the right suprarenal gland. E. The right kidney is firmly attached to the liver.

Vilensky_02.indd 76

Answer is A

Copyright © 2016 Wolters Kluwer

3/17/15 12:22 AM

Abdomen

2.18

1 5 2 3

6

4 7

Vilensky_02.indd 77

3/17/15 12:22 AM

I. LABELS 1. 2. 3. 4. 5. 6. 7.

2.18

Right greater thoracic splanchnic nerve Celiac ganglion; “solar plexus” Left renal vein Right sympathetic trunk and ganglion Left suprarenal (adrenal) gland Left renal artery Left ureter

II. CLINICAL ANATOMY The long left renal vein crosses the midline between SMA and abdominal aorta. Downward pressure on the SMA can compress the vein, producing renal vein entrapment syndrome also known as the nutcracker syndrome. This syndrome may be associated with blood or protein in urine, left abdominal flank pain, nausea, vomiting, and left testicular pain.

Copyright © 2016 Wolters Kluwer

Vilensky_02.indd 78

3/17/15 12:22 AM

Abdomen

2.19

1

6 2 3

7

4 5

Vilensky_02.indd 79

3/17/15 12:22 AM

I. LABELS 1. 2. 3. 4. 5. 6. 7.

2.19

Fibrous capsule of kidney Renal pyramid; terminates in renal papilla Major renal calyx Renal artery Renal pelvis Renal cortex Medulla of kidney

II. QUESTION Which of the following is correct about a patient with a ureteric calculi? A. B. C. D.

The pain is consistently felt in the upper back. The pain is consistently felt in the testes or labia. The pain is due to ischemia of the ureter. The pain is intermittent and may change as the calculi descend down the ureter. E. The flow of urine is always blocked if there is pain.

Vilensky_02.indd 80

Answer is D

Copyright © 2016 Wolters Kluwer

3/17/15 12:22 AM

Abdomen

2.20

1

2

6 7

3

4

5

Vilensky_02.indd 81

3/17/15 12:22 AM

I. LABELS 1. 2. 3. 4. 5. 6. 7.

2.20

Right renal vein Iliohypogastric or ilioinguinal nerve (L1) Genitofemoral nerve (L1 and L2) Ductus deferens Testis and epididymis Inferior mesenteric artery Ureter

II. CLINICAL ANATOMY The branching pattern of the nerves of the lumbar plexus is variable, sometimes making absolute identification of nerves on the posterior abdominal wall confusing. It is best to first find the subcostal nerve directly below the 12th rib and identify the nerves in order from there. Another problem is that the iliohypogastric and ilioinguinal nerves are both part of the ventral ramus of the L1 spinal nerve, and exactly where they split is variable.

Copyright © 2016 Wolters Kluwer

Vilensky_02.indd 82

3/17/15 12:22 AM

Abdomen

2.21

1 3 2

4

5

Vilensky_02.indd 83

3/17/15 12:22 AM

I. LABELS 1. 2. 3. 4. 5.

2.21

Pampiniform plexus of veins Testicular artery Ductus deferens Body of epididymis Parietal layer of tunica vaginalis

II. QUESTION Your patient is complaining about pain on the left side of his scrotum. You examine the patient and notice an enlargement in his left scrotum that disappears when the patient lies down. Upon palpation, the left scrotum feels like “a bag of worms.” Your diagnosis is most likely: Varicocele Testicular torsion Testicular cancer Epididymitis Epididymal cyst

Copyright © 2016 Wolters Kluwer

Vilensky_02.indd 84

Answer is A

A. B. C. D. E.

3/17/15 12:23 AM

Abdomen

2.22

1 7 2 3

4 5 6

Vilensky_02.indd 85

3/17/15 12:23 AM

I. LABELS 1. 2. 3. 4. 5. 6. 7.

2.22

Right kidney Right ureter Inferior vena cava Uterus Round ligament of the uterus Urinary bladder Abdominal aorta

II. CLINICAL ANATOMY An abdominal aortic aneurysm is a localized enlargement of the aorta due to weakness in the wall of the aorta. In a nonobese person, the examiner can feel the pulsations of the dilated vessel on deep palpation of the midabdomen. Rupture of such an aneurysm is associated with severe abdominal pain and likely death because of blood loss.

Copyright © 2016 Wolters Kluwer

Vilensky_02.indd 86

3/17/15 12:23 AM

Abdomen

2.23

1

1

5 6

2 7 3

8 9

4

Vilensky_02.indd 87

3/17/15 12:23 AM

I. LABELS 1. 2. 3. 4. 5. 6. 7. 8. 9.

2.23

Quadratus lumborum Genitofemoral nerve Iliacus External iliac artery Iliohypogastric nerve Ilioinguinal nerve Lateral cutaneous nerve of thigh Obturator nerve Femoral nerve

II. CLINICAL ANATOMY During forced respiration, the quadratus lumborum muscle can assist with inspiration. By preventing the 12th rib from elevating, it allows the diaphragm, which also attaches to the 12th rib, to exert greater downward force on the abdomen, thereby increasing expansion of the thoracic cavity.

Copyright © 2016 Wolters Kluwer

Vilensky_02.indd 88

3/17/15 12:23 AM

Abdomen

2.24

1 4 5

6

2

3

Vilensky_02.indd 89

3/17/15 12:23 AM

I. LABELS 1. 2. 3. 4. 5. 6.

2.24

Right crus of diaphragm Psoas major muscle Median sacral artery Left inferior phrenic artery Muscular part of diaphragm Psoas major muscle

II. QUESTION Your patient has an abscess in the superior part of her thigh that is associated with a muscle that lies lateral to the lumbar vertebrae and attaches to the lesser trochanter of the femur. What muscle is involved in the infection? Quadratus lumborum Transversus abdominis Adductor longus Obturator externus Psoas major

Copyright © 2016 Wolters Kluwer

Vilensky_02.indd 90

Answer is E

A. B. C. D. E.

3/17/15 12:23 AM

Abdomen

2.25

4

5 1 2 6 3

Vilensky_02.indd 91

3/17/15 12:23 AM

I. LABELS 1. 2. 3. 4. 5. 6.

2.25

Right crus of diaphragm Quadratus lumborum muscle Psoas minor muscle Lumbar part of diaphragm Median arcuate ligament Psoas major muscle

II. CLINICAL ANATOMY The arcuate ligaments of the diaphragm are divided into the median, medial, and lateral arcuate ligaments. The aorta passes posterior to the median arcuate ligament, and the celiac trunk arises slightly inferior to that. Median arcuate ligament syndrome involves compression of the celiac trunk by that ligament. The ilioinguinal and iliohypogastric nerves pass posterior to the medial arcuate ligament. The quadratus lumborum muscle and subcostal nerve pass posterior to the lateral arcuate ligament.

Copyright © 2016 Wolters Kluwer

Vilensky_02.indd 92

3/17/15 12:23 AM

Pelvis and Perineum 3.1 1

4 5

2

3 6

Vilensky_03.indd 93

3/17/15 12:24 AM

I. LABELS

3.1

1. Femoral nerve 2. Spermatic cord with genital branch of genitofemoral nerve 3. Great saphenous vein 4. Superficial inguinal ring 5. Ilioinguinal nerve 6. Glans penis

II. CLINICAL ANATOMY The erectile tissue of the penis is composed of the corpus spongiosum and the paired corpora cavernosa. Within the body of the penis, the corpus spongiosum is limited to the ventral aspect and surrounds the penile urethra. But at the head of the penis, the corpus spongiosum expands to form the cone-shaped glans (head) of the penis. At the neck of the glans, the prepuce (foreskin; composed of a double layer of skin) variably covers the glans, and it is this skin that is removed when males are circumcised.

Copyright © 2016 Wolters Kluwer

Vilensky_03.indd 94

3/17/15 12:24 AM

Pelvis and Perineum 3.2 1 3

4 2

5

Vilensky_03.indd 95

3/17/15 12:24 AM

I. LABELS 1. 2. 3. 4. 5.

3.2

Deep dorsal vein of penis (unpaired) Tunica albuginea of corpora cavernosa Dorsal artery of penis (paired) Corpus cavernosum of penis Spongy (penile) urethra

II. CLINICAL ANATOMY Erection of the penis is aided by the fact that the deep dorsal vein of the penis is located deep to the deep (Buck’s) fascia of the penis. As the penis becomes engorged with blood, the deep dorsal vein of the penis is compressed, preventing outflow of blood from the erectile tissues.

Copyright © 2016 Wolters Kluwer

Vilensky_03.indd 96

3/17/15 12:24 AM

Pelvis and Perineum 3.3 1 2

6

3

7

4

5

8

Vilensky_03.indd 97

3/17/15 12:24 AM

I. LABELS 1. 2. 3. 4. 5. 6. 7. 8.

3.3

Corpus spongiosum of penis Corpus cavernosum of penis Ischiocavernosus muscle overlying crus of penis Perineal body External anal sphincter muscle Gracilis muscle Bulbospongiosus muscle Anococcygeal ligament

II. QUESTION Following recovery from a severe automobile crash involving a fracture of the pelvis, your patient is having difficulty with ejaculation during sexual intercourse. Which of the following muscles is most likely paralyzed? Levator ani Bulbospongiosus Obturator internus Gracilis Dartos

Copyright © 2016 Wolters Kluwer

Vilensky_03.indd 98

Answer is B

A. B. C. D. E.

3/17/15 12:24 AM

Pelvis and Perineum 3.4

1

4

5 2 6 3 7

Vilensky_03.indd 99

3/17/15 12:24 AM

I. LABELS

3.4

Bulbospongiosus muscle Perineal branches of pudendal nerve Inferior rectal arteries and nerves Posterior scrotal nerves and superficial perineal arteries 5. Deep transverse perineal muscle 6. External anal sphincter muscle 7. Gluteus maximus muscle

1. 2. 3. 4.

II. CLINICAL ANATOMY The levator ani muscles comprise most of the pelvic diaphragm. The other component is the ischiococcygeus (coccygeus) muscle. The levator ani can be subdivided into the puborectalis, the pubococcygeus, and the iliococcygeus. The puborectalis forms a sling around the anorectal junction and is important for maintaining fecal continence. It relaxes to allow defecation.

Copyright © 2016 Wolters Kluwer

Vilensky_03.indd 100

3/17/15 12:24 AM

Pelvis and Perineum 3.5 1 2 5 3

6

4 7

Vilensky_03.indd 101

3/17/15 12:24 AM

3.5

I. LABELS 1. 2. 3. 4. 5. 6. 7.

Femoral artery Femoral branch of genitofemoral nerve Great saphenous vein Dorsal nerve of penis Testicular artery Ductus deferens Epididymis

II. QUESTION You deliver a child and notice that it has glandular hypospadias. You explain to the parents that: A. The male child’s foreskin is too tight and not easily retracted over the glans. B. The female child is born without a glans clitoris. C. The male child’s external urethral orifice is located on the lower portion of his glans penis. D. The female child’s vaginal frenula are not attached to her glans clitoris. E. The male child’s penis is lacking a glans.

Vilensky_03.indd 102

Answer is C

Copyright © 2016 Wolters Kluwer

3/17/15 12:24 AM

Pelvis and Perineum 3.6

5 1

6

2 3

4

Vilensky_03.indd 103

7

8

3/17/15 12:24 AM

I. LABELS 1. 2. 3. 4. 5. 6. 7. 8.

3.6

External anal sphincter muscle Anal canal Bulb of penis Testis (cut surface) Internal urethral orifice and sphincter Prostate gland Corpus cavernosum of penis Corpus spongiosum of penis

II. QUESTION You are performing a routine digital rectal exam on a 56-year-old male patient and feel an abnormal hardness and roughness through the anterior rectal wall. You suspect: Prostate cancer Benign prostatic hypertrophy Bladder calculus Bladder infection Blockage of ejaculatory duct

Copyright © 2016 Wolters Kluwer

Vilensky_03.indd 104

Answer is A

A. B. C. D. E.

3/17/15 12:24 AM

Pelvis and Perineum 3.7 6 1

7

2 3 8 4 9

5 10

Vilensky_03.indd 105

3/17/15 12:24 AM

I. LABELS 1. 2. 3. 4. 5. 6. 7. 8. 9. 10.

3.7

Urinary bladder Seminal vesicle Ejaculatory duct (proximal portion) Prostate Bulbourethral gland (Cowper’s gland) Ductus deferens Ureter Levator ani muscle Obturator internus muscle Bulb of penis

II. QUESTION A 44-year-old man has an infection in his bulbourethral glands. The duct from this gland empties into the: Bladder Prostate Deep perineal pouch Scrotum Spongy urethra

Copyright © 2016 Wolters Kluwer

Vilensky_03.indd 106

Answer is E

A. B. C. D. E.

3/17/15 12:24 AM

Pelvis and Perineum 3.8 5 1 2

3

6

4

7

Vilensky_03.indd 107

3/17/15 12:24 AM

I. LABELS 1. 2. 3. 4. 5. 6. 7.

3.8

Glans of clitoris Frenulum of clitoris Labium majus Labium minus Prepuce of clitoris External urethral orifice Vaginal orifice

II. CLINICAL ANATOMY Female circumcision (female genital mutilation) is the ritual amputation of some or all of the external female genitalia. The procedures differ among ethnic groups but may include removal of the prepuce, clitoris, and labium majus and minus. Clinical problems resulting from the procedure include recurrent infections, chronic pain, cysts, inability to get pregnant, and complications during childbirth.

Copyright © 2016 Wolters Kluwer

Vilensky_03.indd 108

3/17/15 12:24 AM

Pelvis and Perineum 3.9 3

1

4 2

5 6

Vilensky_03.indd 109

3/17/15 12:24 AM

I. LABELS 1. 2. 3. 4. 5. 6.

3.9

Labium minus Perineal branch of pudendal nerve Mons pubis Greater vestibular gland Superficial transverse perineal muscle Inferior rectal nerves

II. QUESTION A 23-year-old woman comes to the ED claiming that she has lost her tampon in her vagina. Which of the following is the most appropriate course of action? A. Contact the operating room because it is likely that she pushed the tampon through the posterior wall of her vagina into her rectum. B. Perform a rectal exam to locate it. C. Perform a vaginal exam to locate it. D. Have an imaging study done to locate it. E. Dilate her cervix to look for it in her uterus.

Vilensky_03.indd 110

Answer is C

Copyright © 2016 Wolters Kluwer

3/17/15 12:24 AM

Pelvis and Perineum 3.10 5 1 2

6 7

3 4

Vilensky_03.indd 111

3/17/15 12:24 AM

I. LABELS 1. 2. 3. 4. 5. 6. 7.

3.10

Body of clitoris Prepuce of clitoris Labium minus Posterior labial nerve Deep artery of clitoris and dorsal nerve of clitoris Crus of clitoris Bulbospongiosus muscle

II. CLINICAL ANATOMY The body of the clitoris is very variable in size, which is an important fact that is not typically mentioned in anatomy texts. Similar to the penis, it is an erectile organ, but whereas the penis is composed of both corpus spongiosum and cavernosum tissue, the clitoris is only composed of corpus cavernosum. The bulb of the vestibule contains the corpus spongiosum tissue in females.

Copyright © 2016 Wolters Kluwer

Vilensky_03.indd 112

3/17/15 12:24 AM

Pelvis and Perineum 3.11 1 2 3

6 4

7

5

Vilensky_03.indd 113

3/17/15 12:24 AM

I. LABELS 1. 2. 3. 4. 5. 6. 7.

3.11

Median umbilical fold with urachus Urinary bladder Uterus Uterine tube Rectum Ovary Fimbriae of uterine tube

II. QUESTION When performing an ovariectomy, it is necessary to ligate the main blood supply to the ovary prior to removal. The ovarian artery is a branch of the: Abdominal aorta Internal iliac artery External iliac artery Internal pudendal artery Obturator artery

Copyright © 2016 Wolters Kluwer

Vilensky_03.indd 114

Answer is A

A. B. C. D. E.

3/17/15 12:24 AM

Pelvis and Perineum 3.12 5

1 2

6

7

8 3 4

Vilensky_03.indd 115

3/17/15 12:24 AM

I. LABELS 1. 2. 3. 4. 5. 6. 7. 8.

3.12

Common iliac vein Genitofemoral nerve Round ligament of uterus Urinary bladder Ureter Superior hypogastric plexus Rectouterine pouch (of Douglas) Vesicouterine pouch

II. CLINICAL ANATOMY The sympathetic innervation to the pelvic viscera is primarily through the left and right hypogastric nerves, which are derived from lumbar splanchnic nerves. The hypogastric nerves combine with branches from the pelvic splanchnic nerves (parasympathetic) to form plexuses (superior and inferior hypogastric plexuses) to supply the pelvic viscera with autonomic innervation.

Copyright © 2016 Wolters Kluwer

Vilensky_03.indd 116

3/17/15 12:24 AM

Pelvis and Perineum 3.13 1

5

2 3

6

4 7

8

Vilensky_03.indd 117

3/17/15 12:24 AM

3.13

I. LABELS 1. 2. 3. 4. 5. 6. 7. 8.

Internal iliac artery Internal pudendal artery Ureter Inferior hypogastric plexus (pelvic plexus) Lumbosacral trunk Pelvic splanchnic nerves (nervi erigentes) Prostate gland Rectum (reflected)

II. QUESTION After surgery to remove a cancerous prostate gland, the patient no longer can obtain an erection sufficient to achieve penetration for intercourse. It is likely that the surgery damaged which of the following nerves? Pelvic splanchnic Hypogastric Pudendal Genitofemoral S2 dorsal ramus

Copyright © 2016 Wolters Kluwer

Vilensky_03.indd 118

Answer is A

A. B. C. D. E.

3/17/15 12:24 AM

Pelvis and Perineum 3.14

4

1 5 2 3

Vilensky_03.indd 119

6 7

3/17/15 12:24 AM

I. LABELS 1. 2. 3. 4. 5. 6. 7.

3.14

Uterus Vesicouterine pouch Urinary bladder (collapsed) Sacral promontory Rectum Cervix of uterus Vaginal canal

II. QUESTION A 50-year-old woman presents with a mass bulging into the anterior wall of her vagina. When you press on the mass, she feels the urge to urinate. Which of the following is the most likely diagnosis? Rectocele Cystocele Cervical cancer Uterine prolapse Indirect inguinal hernia

Copyright © 2016 Wolters Kluwer

Vilensky_03.indd 120

Answer is B

A. B. C. D. E.

3/17/15 12:24 AM

Pelvis and Perineum 3.15 4

1 2

5

3 6

Vilensky_03.indd 121

3/17/15 12:24 AM

I. LABELS 1. 2. 3. 4. 5. 6.

3.15

Ovary Uterine tube Urinary bladder Small intestine Uterus Urethra

II. QUESTION A 42-year-old multiparous woman is complaining of urinary incontinence when she coughs. Which of the following structures was likely injured during her multiple deliveries resulting in stress incontinence? Obturator internus muscle Piriformis muscle Coccygeus muscle Bulbospongiosus muscle Pubococcygeus muscle

Copyright © 2016 Wolters Kluwer

Vilensky_03.indd 122

Answer is E

A. B. C. D. E.

3/17/15 12:24 AM

Pelvis and Perineum 3.16

1 4 2 5 3

Vilensky_03.indd 123

3/17/15 12:24 AM

I. LABELS 1. 2. 3. 4. 5.

3.16

Urinary bladder Prostate Bulb of penis Obturator internus muscle Prostatic urethra

II. CLINICAL ANATOMY Benign prostatic hypertrophy (BPH) is a condition that affects most men after middle age. The middle (median) lobe of the prostate enlarges and impedes urinary flow in the prostatic urethra. Patients also complain of nocturia (need to urinate at night), dysuria (difficult or painful urination), and urgency (sudden need to void).

Copyright © 2016 Wolters Kluwer

Vilensky_03.indd 124

3/17/15 12:24 AM

Back

4.1 5

1 6 7 2

8

9 10

3

4 11

Vilensky_04.indd 125

3/17/15 12:25 AM

4.1

I. LABELS 1. 2. 3. 4. 5. 6. 7. 8. 9. 10. 11.

Foramen transversarium Vertebral foramen Spinous process Ala of sacrum Anterior tubercle of atlas Posterior tubercle of atlas Dens (odontoid process) of axis Bifid spinous process cervical vertebra Head of rib Tubercle of rib Sacral canal

II. QUESTION A 24-year-old female patient sustained a severe neck injury in an automobile accident. The ED physician is concerned about a dislocation between the atlas and the dens. Which of the following ligaments is primarily responsible for maintaining the integrity of the atlantoaxial joint? Alar ligaments Tectorial membrane Transverse ligament of the atlas Anterior longitudinal ligament Ligamentum flavum

Copyright © 2016 Wolters Kluwer

Vilensky_04.indd 126

Answer is C B

A. B. C. D. E.

3/17/15 12:25 AM

Back

4.2

1

2 3 5 4

6

Vilensky_04.indd 127

3/17/15 12:25 AM

I. LABELS 1. 2. 3. 4. 5. 6.

4.2

Superior articular process Spinous process Inferior articular process Median sacral crest Inferior vertebral notch Auricular surface

II. CLINICAL ANATOMY The inferior articular notch joins with the superior articular notch of the vertebra below to form the intervertebral foramen. In clinical practice, this foramen is typically called the neural foramen. The spinal nerves exit through these foramina. In the thoracic, lumbar, and sacral regions, the nerve exits inferior to the similarly numbered vertebra (i.e., the L4 spinal nerve exits the vertebral column through the L4/L5 intervertebral [neural] foramen).

Copyright © 2016 Wolters Kluwer

Vilensky_04.indd 128

3/17/15 12:25 AM

Back

4.3

1 2 3

4

5 7

6

Vilensky_04.indd 129

3/17/15 12:25 AM

4.3

I. LABELS 1. 2. 3. 4. 5. 6. 7.

Anterior arch of atlas Dens (odontoid process) of axis Intervertebral disk Cerebrospinal fluid in dural sac Spinal cord Trachea Spinous process of C7 (vertebra prominens)

II. QUESTION This radiological image was taken using which of the following modalities? CT Radiographic (x-ray) Ultrasound T1 MRI T2 MRI

Copyright © 2016 Wolters Kluwer

Vilensky_04.indd 130

Answer is E

A. B. C. D. E.

3/17/15 12:25 AM

Back

4.4

4 1

2

3

5

Vilensky_04.indd 131

3/17/15 12:25 AM

I. LABELS 1. 2. 3. 4. 5.

4.4

Body of T12 vertebra Anterior longitudinal ligament Intervertebral disk Vertebral canal containing CSF First sacral vertebra (sacrum)

II. QUESTION If the intervertebral disk identified as number 3 in the image herniates posteriorly, it would most likely impinge on which of the following structures? L2 spinal nerve L3 spinal nerve L4 spinal nerve Ligamentum flavum Filum terminale

Copyright © 2016 Wolters Kluwer

Vilensky_04.indd 132

Answer is C

A. B. C. D. E.

3/17/15 12:25 AM

4.5

Back 5

1 2

3

6

7

4

Vilensky_04.indd 133

3/17/15 12:25 AM

I. LABELS 1. 2. 3. 4. 5. 6. 7.

4.5

Trapezius muscle Rhomboid major muscle Latissimus dorsi muscle Thoracolumbar fascia Splenius capitis muscle Teres major muscle Serratus posterior inferior muscle

II. QUESTION After surgery to remove pathological lymph nodes from a 63-year-old male patient’s neck, the surgeon notices that the patient is no longer able to elevate his shoulder normally. The surgeon assumes that the nerve to the trapezius was inadvertently damaged during surgery. Which of the following nerves was likely injured? Spinal accessory nerve (cranial nerve XI) Dorsal scapular nerve Greater occipital nerve Lesser occipital nerve Auriculotemporal nerve

Copyright © 2016 Wolters Kluwer

Vilensky_04.indd 134

Answer is A B

A. B. C. D. E.

3/17/15 12:25 AM

Back

4.6

1

2

3 6

4

7 8

5

Vilensky_04.indd 135

3/17/15 12:25 AM

I. LABELS 1. 2. 3. 4. 5. 6. 7. 8.

4.6

Galea aponeurotica Greater occipital nerve Semispinalis capitis muscle Superficial branch of transverse cervical artery Rhomboid major muscle Levator scapulae muscle Spinal accessory nerve Serratus posterior superior muscle

II. QUESTION A 35-year-old male patient was brought to the ED after he was “scalped” in an accident in which part of his scalp was removed when it was caught in a piece of farm machinery. Typically, “scalping” occurs through which of the following layers of the scalp? Skin Connective tissue Aponeurosis (galea aponeurotica) Loose areolar connective tissue Periosteum

Copyright © 2016 Wolters Kluwer

Vilensky_04.indd 136

Answer is D B

A. B. C. D. E.

3/17/15 12:25 AM

Back

1

4.7

5

2 6 3

4 7

Vilensky_04.indd 137

3/17/15 12:25 AM

I. LABELS 1. 2. 3. 4. 5. 6. 7.

4.7

Greater occipital nerve Semispinalis capitis muscle Spinal accessory nerve Dorsal rami of spinal nerves Lesser occipital nerve Longissimus cervicis muscle Iliocostalis thoracis muscle

II. CLINICAL ANATOMY The greater occipital nerve is a purely sensory component of the dorsal ramus of the C2 spinal nerve. It supplies sensation to the back of the head. The greater occipital nerve is believed to be a cause of some cervicogenic headaches. The lesser occipital nerve is a branch of the cervical plexus composed of filaments from the ventral rami of C2 and C3. It, too, is purely sensory and supplies the back of the head posterior to the ear.

Copyright © 2016 Wolters Kluwer

Vilensky_04.indd 138

3/17/15 12:25 AM

4.8

Back 1 2 5

3 4

6

Vilensky_04.indd 139

3/17/15 12:25 AM

I. LABELS 1. 2. 3. 4. 5. 6.

4.8

Levator scapulae muscle Iliocostalis cervicis muscle Iliocostalis thoracis muscle Longissimus thoracis muscle Spinalis thoracis muscle Iliocostalis lumborum muscle

II. QUESTION In preparing to do a laminectomy, a surgeon is concerned that she may denervate the iliocostalis lumborum muscle. She is specifically concerned about inadvertently cutting: The The The The The

ventral rami of some lumbar nerves dorsal rami of some lumbar nerves thoracodorsal nerve subcostal nerve lumbar intercostal nerves

Copyright © 2016 Wolters Kluwer

Vilensky_04.indd 140

Answer is B

A. B. C. D. E.

3/17/15 12:25 AM

Back

4.9 3

1 2

4 5 6

7

Vilensky_04.indd 141

3/17/15 12:25 AM

I. LABELS 1. 2. 3. 4. 5. 6. 7.

4.9

Greater occipital nerve Semispinalis cervicis muscle Rectus capitis posterior minor muscle Rectus capitis posterior major muscle Obliquus capitis inferior muscle Levator scapulae muscle Multifidus muscle

II. QUESTION Following a knife wound to the back of the neck, a young female patient is having difficulty retracting her scapulae because of injury to the nerve that innervates the rhomboid major muscle. The ED physician is also concerned about another muscle innervated by this nerve. Which of the following muscles is he concerned about? Multifidus Levator scapulae Semispinalis cervicis Rectus capitis posterior major Obliquus capitis inferior

Copyright © 2016 Wolters Kluwer

Vilensky_04.indd 142

Answer is B

A. B. C. D. E.

3/17/15 12:25 AM

Back

1

4.10

3

2

4

Vilensky_04.indd 143

3/17/15 12:25 AM

I. LABELS 1. 2. 3. 4.

4.10

Levatores costarum muscles Intertransverse lumbar muscles Spinalis muscle Multifidus muscle

II. CLINICAL ANATOMY The spinalis is the most medial component of the erector spinae group of back muscles. It is the least developed of the three, usually appearing only in the thoracic region. The muscle segments arise and insert on spinous processes. Acting unilaterally, the muscle will cause some lateral bending of the spine, whereas acting bilaterally, it will act with the iliocostalis and longissimus to extend the spine.

Copyright © 2016 Wolters Kluwer

Vilensky_04.indd 144

3/17/15 12:25 AM

Back

4.11 3

1

2

Vilensky_04.indd 145

3/17/15 12:25 AM

I. LABELS

4.11

1. Conus medullaris 2. Cauda equina 3. Spinal nerve

II. CLINICAL ANATOMY The conus medullaris is the tapered inferior end of the spinal cord that occurs at approximately L1 or L2. A fine thread of pia mater (filum terminale) anchors the conus medullaris to the coccyx.

Copyright © 2016 Wolters Kluwer

Vilensky_04.indd 146

3/17/15 12:25 AM

Back

4.12 1

2 3

4

Vilensky_04.indd 147

3/17/15 12:25 AM

I. LABELS 1. 2. 3. 4.

4.12

Spinal cord CSF in dural sac Conus medullaris Cauda equina

II. QUESTION Meningitis is suspected in a college-age patient. In order to obtain a sample of the patient’s CSF to test for bacterial infection, where would you most likely insert a needle? Between the occipital bone and the atlas Between C7 and T1 lamina Between L3 and L4 lamina Into the dorsal sacral foramina Into the sacral hiatus

Copyright © 2016 Wolters Kluwer

Vilensky_04.indd 148

Answer is C

A. B. C. D. E.

3/17/15 12:25 AM

Lower Limb

5.1

5

1 6 2 7

3 8

4

Vilensky_05.indd 149

3/17/15 12:26 AM

I. LABELS 1. 2. 3. 4. 5. 6. 7. 8.

5.1

Posterior superior iliac spine Posterior inferior iliac spine Ischial spine Ischial tuberosity Anterior superior iliac spine Anterior inferior iliac spine Lunate surface of acetabulum Acetabular notch

II. CLINICAL ANATOMY The hamstring muscles, which include the long head of biceps femoris, semimembranosus, and semitendinosus, all originate from the ischial tuberosity. A hamstring injury often occurs at this attachment and can be either a strain (“pulled” muscle) or a tear. The muscle may also tear away some of the bony tuberosity, producing an avulsion fracture.

Copyright © 2016 Wolters Kluwer

Vilensky_05.indd 150

3/17/15 12:26 AM

Lower Limb

5.2 2

1

3

4

5

Vilensky_05.indd 151

3/17/15 12:26 AM

I. LABELS 1. 2. 3. 4. 5.

5.2

Auricular surface Superior articular process of sacrum Lateral sacral crest S4 dorsal sacral foramen Coccyx

II. QUESTION A 56-year-old male patient is having radiofrequency ablation of his S4 dorsal ramus for pain. The patient’s pain is likely: In his anterior thigh In his posterior thigh Superficial to his pubic symphysis In his lower buttocks In his penis

Copyright © 2016 Wolters Kluwer

Vilensky_05.indd 152

Answer is D

A. B. C. D. E.

3/17/15 12:26 AM

Lower Limb

5.3

1

2

4

3

5

Vilensky_05.indd 153

3/17/15 12:26 AM

I. LABELS 1. 2. 3. 4. 5.

5.3

Superior articular process of sacrum Sacral promontory S2–S3 ventral sacral foramina Lateral part of sacrum (ala) Coccyx

II. QUESTION A 51-year-old female patient has posterior herniation at her lumbosacral intervertebral disk. Typically, such a condition would most directly compress which of the following spinal nerves? L4 L5 S1 S2 S3

Copyright © 2016 Wolters Kluwer

Vilensky_05.indd 154

Answer is C

A. B. C. D. E.

3/17/15 12:26 AM

Lower Limb

5.4

1 4 5 2

6

3 7

Vilensky_05.indd 155

3/17/15 12:26 AM

I. LABELS 1. 2. 3. 4. 5. 6. 7.

5.4

Sacral canal Arcuate line Anterior inferior iliac spine Site of sacroiliac joint Ischial spine Anterior superior iliac spine Pubic tubercle

II. QUESTION A 37-year-old male patient sustained an avulsion fracture of his anterior inferior iliac spine during an automobile accident. Which muscle is likely most affected? Sartorius Gracilis Tensor fasciae latae Obturator externus Rectus femoris

Copyright © 2016 Wolters Kluwer

Vilensky_05.indd 156

Answer is E

A. B. C. D. E.

3/17/15 12:26 AM

Lower Limb

5.5

1 2 5 3 6 4 7

Vilensky_05.indd 157

3/17/15 12:26 AM

5.5

I. LABELS 1. 2. 3. 4. 5. 6. 7.

Anterior superior iliac spine Iliac fossa Iliopubic eminence Obturator foramen Anterior inferior iliac spine Pubic symphysis Ischial tuberosity

II. QUESTION During labor, a structure that passes through the obturator foramen is sometimes compressed. If this occurs, which of the following is most likely? A. B. C. D.

Weakness in thigh extension Weakness in thigh adduction Decrease in blood to the gluteal muscles Decrease in blood to the quadriceps femoris muscles E. Decrease in blood flow to the iliotibial tract

Vilensky_05.indd 158

Answer is B

Copyright © 2016 Wolters Kluwer

3/17/15 12:26 AM

5.6

Lower Limb 5 1

6

2 3

7

4

8 9

Vilensky_05.indd 159

3/17/15 12:26 AM

I. LABELS 1. 2. 3. 4. 5. 6. 7. 8. 9.

5.6

Femoral neck Lesser trochanter Pectineal line Adductor tubercle Greater trochanter Intertrochanteric crest Linea aspera Lateral epicondyle Intercondylar fossa

II. CLINICAL ANATOMY In patients with osteoporosis, commonly elderly women, the femoral neck is very vulnerable to fracture. This can cause the patient to fall or can occur as a result of a fall. Because the arterial supply to the head of the femur runs along the neck, the arterial vessels are often ruptured during such a fracture. Avascular neurosis of the femoral head may result. This can be treated by replacing the femoral head with a prosthesis (artificial hip), a procedure that was first performed in the United States at the Mayo Clinic in 1969.

Copyright © 2016 Wolters Kluwer

Vilensky_05.indd 160

3/17/15 12:26 AM

Lower Limb

5.7

1

2 3 6 4

7

5

Vilensky_05.indd 161

3/17/15 12:26 AM

I. LABELS 1. 2. 3. 4. 5. 6. 7.

5.7

Iliolumbar ligament Inguinal ligament Sacrospinous ligament Iliofemoral ligament Lesser trochanter Greater trochanter Obturator membrane

II. CLINICAL ANATOMY Meralgia paresthetica is a condition associated with pain or numbness in the lateral thigh caused by compression of the lateral femoral cutaneous nerve (lateral cutaneous nerve of the thigh) as it passes through the inguinal ligament. Compression of the nerve can be due to weight gain, pregnancy, swelling, injury, and tight-fitting clothing. Addressing the cause of the compression usually alleviates the condition, although in some cases, surgery is required to decompress the nerve.

Copyright © 2016 Wolters Kluwer

Vilensky_05.indd 162

3/17/15 12:26 AM

Lower Limb

5.8

1 2

5

3 4

6 7

Vilensky_05.indd 163

3/17/15 12:26 AM

I. LABELS 1. 2. 3. 4. 5. 6. 7.

5.8

Dorsal sacroiliac ligaments Greater sciatic foramen Sacrospinous ligament Sacrotuberous ligament Lesser sciatic foramen Ischiofemoral ligament Pubofemoral ligament

II. QUESTION A mass within the lesser sciatic foramen in a male would likely cause which of the following symptoms? Loss of sensation in the buttocks Decreased blood supply to the penis Weakness in thigh extension Weakness in medial thigh rotation Ischemia to hamstring muscles

Copyright © 2016 Wolters Kluwer

Vilensky_05.indd 164

Answer is B

A. B. C. D. E.

3/17/15 12:26 AM

Lower Limb

5.9

4 5 1 2

6

7 3

Vilensky_05.indd 165

3/17/15 12:26 AM

I. LABELS 1. 2. 3. 4. 5. 6. 7.

5.9

Piriformis muscle Inferior cluneal nerve Sciatic nerve Superior gluteal artery Gluteus minimus muscle Posterior femoral cutaneous nerve Quadratus femoris muscle

II. CLINICAL ANATOMY Piriformis syndrome is a neuromuscular disorder that occurs when the piriformis muscle compresses the sciatic nerve. Piriformis syndrome usually begins with pain, tingling, or numbness within the area of the buttocks. Pain can be severe and radiate along the sciatic nerve (sciatica). The compression of the nerve often occurs after long periods of sitting as in a prolonged car trip. Pain may also be associated with climbing stairs or running.

Copyright © 2016 Wolters Kluwer

Vilensky_05.indd 166

3/17/15 12:26 AM

5.10

Lower Limb 4 5 1

2

6

7

8

3

Vilensky_05.indd 167

3/17/15 12:26 AM

I. LABELS 1. 2. 3. 4. 5. 6. 7. 8.

5.10

Sacrotuberous ligament Tibial nerve Medial head of gastrocnemius muscle Gluteus medius muscle Piriformis muscle Inferior gluteal artery Common fibular (peroneal) nerve Lateral sural cutaneous nerve

II. QUESTION While doing some home renovations, a 42-year-old female fell on a windowpane and sustained a deep laceration to her left buttock. Following her injury, she walks with a limp because her right hip drops when her weight is on her left foot (during left stance phase). To compensate, she leans to the left when she takes a step with her right foot; this is known as Trendelenburg gait. The ED physician assumes there is damage to the nerve that innervates the gluteus medius and minimus muscles. Which nerve was lesioned? Left inferior gluteal nerve Right inferior gluteal nerve Left superior gluteal nerve Right superior gluteal nerve Left obturator nerve

Copyright © 2016 Wolters Kluwer

Vilensky_05.indd 168

Answer is C

A. B. C. D. E.

3/17/15 12:26 AM

5.11

Lower Limb 1

4

2

3

5

6 7

8

Vilensky_05.indd 169

3/17/15 12:26 AM

5.11

I. LABELS 1. 2. 3. 4. 5. 6. 7. 8.

Piriformis muscle Obturator internus muscle Adductor minimus muscle Gluteus medius muscle Adductor magnus muscle (adductor portion) Short head of biceps femoris muscle Long head of biceps femoris muscle (cut) Plantaris muscle

II. QUESTION A surgeon is about to repair a wound to a patient’s adductor magnus muscle and asks the anesthesiologist to be sure to eliminate any activity in all the nerves that innervate this muscle. Which of the following nerves must be anesthetized? Obturator and femoral Femoral and tibial part of sciatic Femoral and pudendal Obturator and tibial part of sciatic Tibial part of sciatic and saphenous

Copyright © 2016 Wolters Kluwer

Vilensky_05.indd 170

Answer is D

A. B. C. D. E.

3/17/15 12:26 AM

5.12

Lower Limb 1

5

6 2

3

7

4

Vilensky_05.indd 171

3/17/15 12:26 AM

I. LABELS 1. 2. 3. 4. 5. 6. 7.

5.12

Gluteus maximus muscle Semitendinosus muscle Tibial nerve Medial head of gastrocnemius muscle Iliotibial tract Long head of biceps femoris muscle Common fibular (peroneal) nerve

II. CLINICAL ANATOMY The common fibular (peroneal) nerve is the most commonly injured nerve in the body. As it winds around the neck of the fibula, it is subcutaneous and vulnerable to injury. Following a severe compression of the common fibular nerve, the patient exhibits a foot drop because all of the dorsiflexors of the foot are paralyzed. As the patient walks, the foot hits the ground sharply with a definite sounding “clop.” The patient must also lift the foot higher during the swing phase of walking to prevent the toes from dragging on the ground. This distinctive walking pattern is known as steppage gait.

Copyright © 2016 Wolters Kluwer

Vilensky_05.indd 172

3/17/15 12:26 AM

5.13

Lower Limb 1 2

7

3 4

8

5

6 9

Vilensky_05.indd 173

3/17/15 12:26 AM

I. LABELS 1. 2. 3. 4. 5. 6. 7. 8. 9.

5.13

Inguinal ligament Femoral nerve Femoral artery Sartorius muscle Rectus femoris muscle Anterior cutaneous branches of femoral nerve Femoral vein Great saphenous vein Gracilis muscle

II. CLINICAL ANATOMY Varicose veins are twisted, enlarged, subcutaneous veins that result from an insufficiency in the valves of these veins. The great saphenous vein is very often affected by this condition, which is disfiguring and sometimes painful. Surprisingly, removing or closing the great saphenous vein using a laser ablation process causes no ill effects and typically alleviates the condition as the blood finds other routes to return to the heart.

Copyright © 2016 Wolters Kluwer

Vilensky_05.indd 174

3/17/15 12:26 AM

5.14

Lower Limb 1

2

3

4 5

6

Vilensky_05.indd 175

3/17/15 12:26 AM

I. LABELS

5.14

1. Lateral femoral cutaneous nerve (lateral cutaneous nerve of thigh) 2. Lateral circumflex femoral artery 3. Medial circumflex femoral artery 4. Adductor longus muscle 5. Obturator artery and nerve 6. Gracilis muscle

II. QUESTION Following a fracture of the neck of the femur in an elderly woman, you are concerned about avascular necrosis of the femoral head. Which artery supplies the femoral head and, if severed, would cause avascular necrosis? Medial femoral circumflex Obturator Deep circumflex iliac Superficial circumflex iliac Internal pudendal

Copyright © 2016 Wolters Kluwer

Vilensky_05.indd 176

Answer is A

A. B. C. D. E.

3/17/15 12:26 AM

Lower Limb

5.15

1 2

3 4 6

5 7

8

Vilensky_05.indd 177

3/17/15 12:26 AM

I. LABELS 1. 2. 3. 4. 5. 6. 7. 8.

5.15

Tensor fasciae latae muscle Iliopsoas muscle Sartorius muscle Lateral circumflex femoral artery Iliotibial tract Adductor longus muscle Rectus femoris muscle Vastus medialis muscle

II. QUESTION A 53-year-old male patient is very unsteady due to a lack of tension in his iliotibial tract. What two muscles can tense this structure? Gluteus medius and minimus Gluteus maximus and medius Gluteus maximus and tensor fascia latae Tensor fascia latae and rectus femoris Tensor fascia latae and gluteus medius

Copyright © 2016 Wolters Kluwer

Vilensky_05.indd 178

Answer is C

A. B. C. D. E.

3/17/15 12:26 AM

5.16

Lower Limb

3

4 1

5

6 7 2

Vilensky_05.indd 179

3/17/15 12:26 AM

I. LABELS

5.16

1. Descending branch of lateral femoral circumflex artery 2. Rectus femoris muscle (cut) 3. Deep artery of thigh (deep femoral artery) 4. Adductor brevis muscle 5. Anterior branch of obturator nerve 6. Femoral artery (cut) 7. Gracilis muscle

II. CLINICAL ANATOMY The femoral artery is the continuation of the external iliac artery after it crosses deep to the inguinal ligament. It is located between the femoral nerve and vein in the femoral triangle and descends in the adductor (subsartorial or Hunter’s) canal before passing through the adductor hiatus, at which point it becomes the popliteal artery. The femoral artery supplies the muscles of the anterior thigh.

Copyright © 2016 Wolters Kluwer

Vilensky_05.indd 180

3/17/15 12:26 AM

5.17

Lower Limb 6 1 7 2 8 3

9 4

10

5

Vilensky_05.indd 181

3/17/15 12:26 AM

I. LABELS 1. 2. 3. 4. 5. 6. 7. 8. 9. 10.

5.17

Iliopsoas muscle Tensor fasciae latae muscle Rectus femoris muscle Vastus lateralis muscle Patellar ligament Inguinal ligament Pectineus muscle Adductor longus muscle Gracilis muscle Vastus medialis muscle

II. QUESTION The gracilis muscle is commonly harvested for reconstructive surgery in a damaged part of the body. The harvested free flap includes segments of the nerve and primary artery that supply this muscle. These are the: A. Obturator nerve and medial femoral circumflex artery B. Obturator nerve and profunda femoris artery C. Obturator nerve and obturator artery D. Saphenous nerve and obturator artery E. Femoral nerve and obturator artery

Vilensky_05.indd 182

Answer is C

Copyright © 2016 Wolters Kluwer

3/17/15 12:26 AM

Lower Limb

5.18

1 5 2 6 3 4

Vilensky_05.indd 183

7

3/17/15 12:26 AM

I. LABELS 1. 2. 3. 4. 5. 6. 7.

5.18

Pyramidalis muscle Iliopsoas muscle Sciatic nerve and accompanying artery Gluteus maximus muscle Sartorius muscle Obturator internus muscle Rectum

II. QUESTION The patient with tuberculosis presents with a mass immediately below the inguinal ligament. There is pain upon extension of the thigh. As part of the differential diagnosis, you would likely include an abscess of which of the following muscles that inserts into the lesser trochanter of the femur? Sartorius Iliopsoas Gluteus maximus Obturator externus Rectus femoris

Copyright © 2016 Wolters Kluwer

Vilensky_05.indd 184

Answer is B

A. B. C. D. E.

3/17/15 12:26 AM

Lower Limb 1

5.19 4 5

2

3

6

7 8

Vilensky_05.indd 185

3/17/15 12:26 AM

I. LABELS 1. 2. 3. 4. 5. 6. 7. 8.

5.19

Rectus femoris muscle Vastus intermedius muscle Vastus lateralis muscle Sartorius muscle Great saphenous vein Femoral artery and vein Gracilis muscle Adductor magnus muscle

II. QUESTION A 9-year-old male child with cerebral palsy has a spastic adductor magnus muscle, and the surgeon is going to transplant its most distal insertion to a more proximal location to reduce the child’s spasticity. The muscle’s normal distal insertion is on the adductor tubercle, which is located on the: Proximal tibial Proximal fibula Mid lateral femur Distal medial femur Distal lateral femur

Copyright © 2016 Wolters Kluwer

Vilensky_05.indd 186

Answer is D

A. B. C. D. E.

3/17/15 12:26 AM

Lower Limb

5.20

1 2 4 5 6

7 3

Vilensky_05.indd 187

3/17/15 12:26 AM

I. LABELS 1. 2. 3. 4. 5. 6. 7.

5.20

Rectus femoris muscle Vastus intermedius muscle Biceps femoris muscle Sartorius muscle Femoral artery and vein Great saphenous vein Gracilis muscle

II. CLINICAL ANATOMY The action of the sartorius muscle is that required to sit cross-legged as tailors historically did. The muscle flexes and laterally rotates the thigh and flexes the leg. It inserts on the proximal medial shaft of the tibia as part of the “pes anserinus,” with the gracilis and semitendinosus. The sartorius muscle is innervated by the femoral nerve.

Copyright © 2016 Wolters Kluwer

Vilensky_05.indd 188

3/17/15 12:26 AM

5.21

Lower Limb 1

4 5

2

3 6

Vilensky_05.indd 189

3/17/15 12:26 AM

I. LABELS 1. 2. 3. 4. 5. 6.

5.21

Medial condyle of tibia Soleal line Medial malleolus Lateral tibial condyle Head of fibula Lateral malleolus

II. CLINICAL ANATOMY The distal joint between the fibula and the tibia is one of the few syndesmoses (fibrous joints, in contrast to synovial joints) in the human body. The distal tibiofibular syndesmosis helps to maintain proper alignment between the distal ends of the tibia and fibula. Injury to the distal tibiofibular syndesmosis is commonly called a high ankle sprain. If left untreated, high ankle sprains can lead to instability of the entire ankle joint.

Copyright © 2016 Wolters Kluwer

Vilensky_05.indd 190

3/17/15 12:26 AM

Lower Limb

5.22

1

4 5 2

6

3

Vilensky_05.indd 191

3/17/15 12:26 AM

5.22

I. LABELS 1. 2. 3. 4. 5. 6.

Tibial tuberosity Medial condyle of tibia Posterior intercondylar area Anterior intercondylar area Lateral condyle of tibia Medial and lateral intercondylar tubercles

II. QUESTION A 15-year-old male patient has been diagnosed with Osgood-Schlatter disease, which is characterized by inflammation of the patellar ligament. This diagnosis can be confirmed radiographically by looking at the: Tibial tuberosity Medial condyle of tibia Lateral condyle of tibia Medial intercondylar tubercle Lateral intercondylar tubercle

Copyright © 2016 Wolters Kluwer

Vilensky_05.indd 192

Answer is A

A. B. C. D. E.

3/17/15 12:26 AM

Lower Limb

5.23 4

1 2 5 3

Vilensky_05.indd 193

3/17/15 12:26 AM

I. LABELS 1. 2. 3. 4. 5.

5.23

Medial meniscus of knee joint Anterior cruciate ligament Posterior cruciate ligament Patellar tendon Lateral meniscus of knee joint

II. QUESTION A 21-year-old male football player sustained a knee injury during a tackle. You notice that there is excessive anterior displacement of the tibia relative to the femur (positive anterior drawer sign). This likely indicates a tear in which of the following structures? Anterior cruciate ligament Posterior cruciate ligament Lateral collateral ligament Patellar tendon Lateral meniscus

Copyright © 2016 Wolters Kluwer

Vilensky_05.indd 194

Answer is A

A. B. C. D. E.

3/17/15 12:26 AM

Lower Limb

1

5.24

3

2 4

5

Vilensky_05.indd 195

3/17/15 12:26 AM

I. LABELS 1. 2. 3. 4. 5.

5.24

Medial condyle of femur Posterior cruciate ligament Lateral condyle of femur Lateral (fibular) collateral ligament Head of fibula

II. CLINICAL ANATOMY The medial and lateral collateral ligaments are taut when the knee is extended, thus providing stability to the knee joint during standing. The lateral (fibular) collateral ligament is thinner and more rounded than the medial collateral ligament and extends from the lateral epicondyle of the femur to the head of the fibula. Unlike the medial collateral ligament, it is not attached to the joint capsule or the ipsilateral meniscus. Because of this, the lateral collateral ligament is less susceptible to injury than the medial collateral ligament.

Copyright © 2016 Wolters Kluwer

Vilensky_05.indd 196

3/17/15 12:26 AM

Lower Limb

5.25

1

2

5 3 6 4

Vilensky_05.indd 197

3/17/15 12:26 AM

I. LABELS 1. 2. 3. 4. 5. 6.

5.25

Shaft of femur Patella Infrapatellar fat pad Patellar tendon Anterior cruciate ligament Posterior cruciate ligament

II. CLINICAL ANATOMY Chondromalacia patellae is a painful condition associated with inflammation of the internal surface of the patella and softening of the cartilage located there. This condition is also referred to as patellofemoral pain syndrome. The condition is most common among young individuals who are actively involved in sports, but it may also affect older individuals who overuse their knees.

Copyright © 2016 Wolters Kluwer

Vilensky_05.indd 198

3/17/15 12:26 AM

Lower Limb

5.26

1

2

6

3

7

8

4 5

Vilensky_05.indd 199

3/17/15 12:26 AM

I. LABELS 1. 2. 3. 4. 5. 6. 7. 8.

5.26

Semitendinosus muscle Semimembranosus muscle Medial superior genicular artery Deep fascia overlying popliteus insertion on tibia Soleus muscle Popliteal artery Common fibular (peroneal) nerve Plantaris muscle

II. CLINICAL ANATOMY The plantaris muscle is a thin muscle originating from the lateral supracondylar line of the femur; it becomes a narrow tendon that descends between the gastrocnemius and soleus muscles to attach to the calcaneal (Achilles) tendon or to the medial side of the tubercle of the calcaneus. The plantaris tendon may be harvested for tendon grafts in other areas of the body and is susceptible to injury during activities that involve forced dorsiflexion of the ankle with the knee extended. Plantaris tendon rupture is sometimes referred to as tennis leg.

Copyright © 2016 Wolters Kluwer

Vilensky_05.indd 200

3/17/15 12:26 AM

5.27

Lower Limb 3

4

5

1

6

2

Vilensky_05.indd 201

3/17/15 12:26 AM

I. LABELS

5.27

1. Tendons of tibialis posterior, flexor digitorum longus, and flexor hallucis longus muscles (from anterior to posterior) 2. Tendon of tibialis anterior muscle 3. Tendon of gracilis muscle 4. Medial head of gastrocnemius muscle 5. Soleus muscle 6. Calcaneal or Achilles tendon

II. QUESTION A 37-year-old female was involved in an automobile accident in which she sustained traumatic injury to her knee. Upon examination, she appears to have a complete tibial nerve transection. Which of the following muscles would still be able to provide some ankle plantarflexion despite this injury? Tibialis posterior Tibialis anterior Flexor digitorum longus Fibularis (peroneus) brevis Flexor hallucis longus

Copyright © 2016 Wolters Kluwer

Vilensky_05.indd 202

Answer is D

A. B. C. D. E.

3/17/15 12:26 AM

5.28

Lower Limb 4

5

1

6

7

2 3

Vilensky_05.indd 203

3/17/15 12:26 AM

I. LABELS

5.28

Tendinous arch of soleus muscle Tibial nerve Posterior tibial artery Biceps femoris muscle Muscular branch of tibial nerve to lateral head of gastrocnemius muscle 6. Fibular (peroneal) artery 7. Flexor hallucis longus muscle

1. 2. 3. 4. 5.

II. CLINICAL ANATOMY The gastrocnemius arises from the femoral condyles so that although it is primarily a plantarflexor of the ankle, it can also flex the knee. The soleus arises from both the fibula and tibia and thus is only an ankle plantarflexor. Both are innervated by the tibial nerve and both insert onto the calcaneus via the calcaneal (Achilles) tendon. The soleus can be functionally isolated for physical examination by having the patient fully flex the knee.

Copyright © 2016 Wolters Kluwer

Vilensky_05.indd 204

3/17/15 12:26 AM

Lower Limb

5.29 6

1

2

7

3

4 5

Vilensky_05.indd 205

8

3/17/15 12:26 AM

I. LABELS 1. 2. 3. 4. 5. 6. 7. 8.

5.29

Lateral head of gastrocnemius muscle Soleus muscle Fibularis (peroneus) brevis muscle Tendon of fibularis (peroneus) longus muscle Extensor digitorum brevis muscle Patellar ligament Tibialis anterior muscle Tendons of extensor digitorum longus muscle

II. QUESTION A 67-year-old woman sustained an avulsion fracture of the tuberosity of the fifth metatarsal (dancer’s fracture) when she forcefully inverted her foot while walking down some stairs. Which of the following muscles attaches to the tuberosity of the fifth metatarsal? Fibularis longus Fibularis brevis Fibularis tertius Tibialis anterior Tibialis posterior

Copyright © 2016 Wolters Kluwer

Vilensky_05.indd 206

Answer is B

A. B. C. D. E.

3/17/15 12:26 AM

5.30

Lower Limb 6

1 2

5 3

4

6 7

Vilensky_05.indd 207

3/17/15 12:26 AM

I. LABELS 1. 2. 3. 4. 5. 6.

5.30

Tibialis anterior muscle Tibia Peroneus (fibularis) longus and brevis muscles Soleus muscle Posterior tibial artery and vein and tibial nerve Small saphenous vein and sural nerve

II. CLINICAL ANATOMY Whereas the small saphenous vein arises from the dorsal venous arch of the foot and passes posterior to the lateral malleolus, the great saphenous vein arises from the medial side of the arch and passes anterior to the medial malleolus. Because it can be always be found there, the large saphenous vein at this location is used for IV injections when more commonly used veins such as the median cubital vein are not available.

Copyright © 2016 Wolters Kluwer

Vilensky_05.indd 208

3/17/15 12:27 AM

5.31

Lower Limb 6

1

2 3 4

7

5

Vilensky_05.indd 209

3/17/15 12:27 AM

5.31

I. LABELS 1. 2. 3. 4. 5. 6. 7.

Tibialis anterior muscle Superior extensor retinaculum Inferior extensor retinaculum Tendon of fibularis (peroneus) tertius muscle Tendons of extensor digitorum longus muscle Patellar ligament Extensor hallucis brevis muscle

II. QUESTION A 47-year-old male presents with severe pain along the medial aspect of his anterior leg. The patient is overweight and recently started walking several miles a day after being sedentary for a number of years. Which of the following is the most likely cause of his pain? A. Stress fracture of his fibula B. Shin splints associated with tendonitis of his tibialis anterior C. Shin splints associated with tendonitis of his fibularis (peroneus) longus D. Osgood-Schlatter disease E. Tibial nerve entrapment

Vilensky_05.indd 210

Answer is B

Copyright © 2016 Wolters Kluwer

3/17/15 12:27 AM

5.32

Lower Limb

1

2 3

6

4

7

5

Vilensky_05.indd 211

3/17/15 12:27 AM

5.32

I. LABELS 1. 2. 3. 4. 5. 6. 7.

Flexor digitorum longus muscle Flexor retinaculum Tendon of tibialis posterior muscle Abductor hallucis muscle Tendon of flexor hallucis longus muscle Tendon of flexor digitorum longus muscle Quadratus plantae muscle

II. QUESTION A 53-year-old male patient presents with complaints about movements in his foot. Upon examination, you notice that when flexing digits 2 to 5, the toes seem to flex obliquely instead of flexing normally (parallel to the long axis of the foot). An injury to which of the following muscles would cause this abnormal flexion? Flexor digitorum brevis Extensor digitorum brevis Lumbricals of all the digits Quadratus plantae Fibularis (peroneus) longus

Copyright © 2016 Wolters Kluwer

Vilensky_05.indd 212

Answer is D

A. B. C. D. E.

3/17/15 12:27 AM

Lower Limb

5.33

1

2 3

5 6 7

4

8

Vilensky_05.indd 213

3/17/15 12:27 AM

I. LABELS 1. 2. 3. 4. 5. 6. 7. 8.

5.33

First metatarsal bone Intermediate cuneiform bone Navicular bone Trochlea of talus Tuberosity of fifth metatarsal Cuboid bone Tarsal sinus Calcaneal tuberosity

II. CLINICAL ANATOMY The tarsal sinus is a hollow or canal formed by the groove of the talus and the interosseous groove of the calcaneus; the talocalcaneal interosseous ligament is found within the sinus. A patient may feel pain in the area of the sinus (tarsal sinus syndrome), which is usually associated with a lateral sprain of the ankle.

Copyright © 2016 Wolters Kluwer

Vilensky_05.indd 214

3/17/15 12:27 AM

Lower Limb

5.34

3 1

4 5 2

Vilensky_05.indd 215

3/17/15 12:27 AM

I. LABELS 1. 2. 3. 4. 5.

5.34

Medial malleolus Sustentaculum tali Position of ankle joint Lateral malleolus Position of subtalar joint

II. CLINICAL ANATOMY The subtalar joint is a complex joint between the calcaneus and the talus. It is also known as the talocalcaneal joint. It is actually three separate articulations between the talus and calcaneus. The movements that occur at this joint are inversion and eversion and pronation and supination. When you twist your upper body while standing and look over your shoulder, the subtalar joints of your feet move with the rest of your body. If you look over your left shoulder, the left foot supinates whereas the right pronates. Instability at the subtalar joint can occur after a severe ankle injury.

Copyright © 2016 Wolters Kluwer

Vilensky_05.indd 216

3/17/15 12:27 AM

Lower Limb

5.35

1

4 5 2 6

3

Vilensky_05.indd 217

3/17/15 12:27 AM

5.35

I. LABELS Tuberosity of fifth metatarsal Long plantar ligament Calcaneus Navicular bone Plantar calcaneonavicular ligament (spring ligament) 6. Sustentaculum tali

1. 2. 3. 4. 5.

II. QUESTION A 27-year-old woman is complaining of pain in her foot associated with a sprain of her spring ligament. Which muscle is responsible for providing support to the inferior surface of this ligament? Tibialis posterior Fibularis (peroneus) longus Fibularis (peroneus) brevis Flexor hallucis longus Flexor hallucis brevis

Copyright © 2016 Wolters Kluwer

Vilensky_05.indd 218

Answer is A

A. B. C. D. E.

3/17/15 12:27 AM

Lower Limb

5.36

3

1

4

2

5

6

Vilensky_05.indd 219

3/17/15 12:27 AM

I. LABELS 1. 2. 3. 4. 5. 6.

5.36

Cuboid bone Lateral cuneiform bone Talus Navicular bone Medial cuneiform bone Dorsal interossei muscles

II. CLINICAL ANATOMY The five tarsal bones that comprise the midfoot are the navicular; cuboid; and the medial, intermediate, and lateral cuneiform bones. The midtarsal joint is a functional rather than an anatomical joint that consists of the calcaneocuboid and talonavicular joints, which act together with the subtalar joint when the foot is inverted and everted.

Copyright © 2016 Wolters Kluwer

Vilensky_05.indd 220

3/17/15 12:27 AM

Lower Limb

5.37

1

2 5

3

4

Vilensky_05.indd 221

6

3/17/15 12:27 AM

I. LABELS 1. 2. 3. 4. 5. 6.

5.37

Sural nerve Small saphenous vein Tendon of peroneus (fibularis) brevis muscle Tendons of extensor digitorum longus muscle Branches of superficial fibular (peroneal) nerve Deep fibular (peroneal) nerve

II. QUESTION A patient sustained an injury to his upper lateral leg and you wish to test the integrity of the deep fibular (peroneal) nerve. You would: A. Test for sensation along the lateral aspect of the leg B. Test for sensation over the fifth metatarsal C. Test for sensation in the web space between the first and second toes D. Test for activity in the fibularis (peroneus) longus E. Test for activity in the fibularis (peroneus) brevis

Vilensky_05.indd 222

Answer is C

Copyright © 2016 Wolters Kluwer

3/17/15 12:27 AM

Lower Limb

5.38

1

4 2

5

3

Vilensky_05.indd 223

3/17/15 12:27 AM

I. LABELS 1. 2. 3. 4. 5.

5.38

Inferior extensor retinaculum Tendons of extensor digitorum longus muscle Dorsal metatarsal arteries Tendon of extensor hallucis longus muscle Deep plantar artery

II. QUESTION A 43-year-old woman is complaining of cold feet, and an imaging study shows decreased blood flow in her plantar arch. This arch is primarily supplied by blood from: A. The medial and lateral plantar arteries B. The lateral plantar artery and the deep branch of the dorsalis pedis artery C. The medial plantar and perforating arteries D. The dorsalis pedis and perforating arteries E. The first second plantar metatarsal arteries

Vilensky_05.indd 224

Answer is B

Copyright © 2016 Wolters Kluwer

3/17/15 12:27 AM

5.39

Lower Limb

4 5 1 2

6

3 7

Vilensky_05.indd 225

3/17/15 12:27 AM

I. LABELS 1. 2. 3. 4. 5. 6. 7.

5.39

Flexor digiti minimi brevis muscle Flexor digitorum brevis muscle Abductor digiti minimi muscle Tendon of flexor hallucis longus muscle Flexor hallucis brevis muscle Abductor hallucis muscle Plantar aponeurosis (plantar fascia; cut)

II. CLINICAL ANATOMY Plantar fasciitis is a common painful condition of the heel and plantar surface of the foot. It is characterized by inflammation, fibrosis, or structural degeneration of the plantar aponeurosis, which originates on the medial tubercle of the calcaneus and extends to the toes. The classic presentation of plantar fasciitis is sharp medial heel pain that is made worse by bearing weight on the heel. It is typically especially painful when getting out of bed and during walking after long periods of rest. Although for most people this is a temporary condition that can be treated with rest and stretching exercises, in some patients, it becomes chronic and can persist for years.

Copyright © 2016 Wolters Kluwer

Vilensky_05.indd 226

3/17/15 12:27 AM

Lower Limb

1 2

5.40

4 5

6 3

Vilensky_05.indd 227

3/17/15 12:27 AM

I. LABELS 1. 2. 3. 4. 5. 6.

5.40

Transverse head adductor hallucis muscle Interossei muscles Tendon of peroneus (fibularis) longus muscle Flexor hallucis brevis muscle Oblique head of adductor hallucis muscle Abductor hallucis muscle (cut)

II. QUESTION A 68-year-old male patient shows atrophy in the space between his first and second metatarsals. Which of the following muscles is most likely to be atrophic? First plantar interosseous First dorsal interosseous Second plantar interosseous Second dorsal interosseous Oblique head of adductor hallucis

Copyright © 2016 Wolters Kluwer

Vilensky_05.indd 228

Answer is B

A. B. C. D. E.

3/17/15 12:27 AM

5.41

Lower Limb 5

6 1

2

7

3

4

Vilensky_05.indd 229

3/17/15 12:27 AM

I. LABELS 1. 2. 3. 4. 5. 6. 7.

5.41

Common plantar digital nerves Flexor digitorum brevis muscle Lateral plantar artery and nerve Abductor digiti minimi muscle Proper plantar digital artery Lumbrical muscles Medial plantar nerve

II. CLINICAL ANATOMY The pedal lumbrical muscles all arise on the medial side of respective tendons II to V of the flexor digitorum longus. They act primarily to flex the metatarsophalangeal joints of the associated digits. The first lumbrical is innervated by the medial plantar nerve, whereas the other three are innervated by the lateral plantar nerve. Injury to the lateral plantar nerve would affect digits 2 through 5 but not the hallux.

Copyright © 2016 Wolters Kluwer

Vilensky_05.indd 230

3/17/15 12:27 AM

Lower Limb

5.42

1

2 4

3

5

Vilensky_05.indd 231

3/17/15 12:27 AM

5.42

I. LABELS 1. 2. 3. 4. 5.

Flexor digitorum brevis muscle Tendons of flexor digitorum longus muscle Quadratus plantae muscle Medial plantar artery Abductor hallucis muscle

II. QUESTION A surgeon is about to operate on the hallux and wishes to stop blood flowing in the medial plantar artery. In order to do so, he would ask the surgical assistant to compress the: Anterior tibial artery Posterior tibial artery Perforating artery Dorsalis pedis artery Medial tarsal artery

Copyright © 2016 Wolters Kluwer

Vilensky_05.indd 232

Answer is B

A. B. C. D. E.

3/17/15 12:27 AM

Upper Limb

6.1 4

1 5

2

6

3

Vilensky_06.indd 233

3/17/15 12:29 AM

I. LABELS 1. 2. 3. 4. 5. 6.

6.1

Acromion Head of humerus Surgical neck of humerus Clavicle Position of acromioclavicular joint Coracoid process

II. QUESTION A football player landed hard on his shoulder and was diagnosed with a separated shoulder. Which of the following best describes a separated shoulder? A. A separation between the humeral head and glenoid fossa B. A separation between the sternal end of the clavicle and the sternum C. A separation between the acromial process of the scapula and the distal (acromial) end of the clavicle D. A separation between glenoid process and body of scapula E. A separation between the coracoid process and the body of the scapula

Vilensky_06.indd 234

Answer is C

Copyright © 2016 Wolters Kluwer

3/17/15 12:29 AM

Upper Limb

6.2 6

1

2 3

4 7 5

8

Vilensky_06.indd 235

3/17/15 12:29 AM

I. LABELS 1. 2. 3. 4. 5. 6. 7. 8.

6.2

Scapular notch Supraspinous fossa Scapular spine Infraspinous fossa Medial border of scapula Acromion Lateral border of scapula Inferior angle of scapula

II. CLINICAL ANATOMY Movements of the scapula are essential for free movement of the upper limb. In order to fully abduct the arm overhead, the glenoid fossa of the scapula must be rotated superiorly (upward rotation). When the glenoid fossa rotates inferiorly, this is downward rotation. The entire scapula can also move up and down; this is known as scapular elevation and depression.

Copyright © 2016 Wolters Kluwer

Vilensky_06.indd 236

3/17/15 12:29 AM

6.3

Upper Limb 1 4 2

5 3

Vilensky_06.indd 237

6

3/17/15 12:29 AM

I. LABELS 1. 2. 3. 4. 5. 6.

6.3

Greater tubercle Intertubercular sulcus Capitulum Lesser tubercle Coronoid fossa Medial epicondyle

II. QUESTION After a shoulder injury, the patient is diagnosed with a dislocation of the tendon that traverses the intertubercular sulcus of the humerus. Which of the following tendons is most likely dislocated? Tendon of long head of triceps brachii Tendon of lateral head of triceps brachii Tendon of medial head of triceps brachii Tendon of short head of biceps brachii Tendon of long head of biceps brachii

Copyright © 2016 Wolters Kluwer

Vilensky_06.indd 238

Answer is E

A. B. C. D. E.

3/17/15 12:29 AM

Upper Limb

6.4

4

5 1

6

7 2

3

Vilensky_06.indd 239

3/17/15 12:29 AM

I. LABELS 1. 2. 3. 4. 5. 6. 7.

6.4

Trapezius muscle Rhomboid major muscle Latissimus dorsi muscle Spine of scapula Deltoid muscle Infraspinous fascia covering infraspinatus muscle Teres major muscle

II. QUESTION After extensive neck surgery to remove a tumor from the right posterior triangle of neck, the patient shows weakness in shoulder abduction and scapular retraction. The nerve innervating which of the following muscles was most likely injured? Pectoralis major Middle deltoid Coracobrachialis Trapezius Rhomboid major

Copyright © 2016 Wolters Kluwer

Vilensky_06.indd 240

Answer is D

A. B. C. D. E.

3/17/15 12:29 AM

Upper Limb

6.5

4

1 2

5

3

6

Vilensky_06.indd 241

3/17/15 12:29 AM

I. LABELS 1. 2. 3. 4. 5. 6.

6.5

Suprascapular artery Omohyoid muscle Suprascapular nerve Splenius capitis muscle Accessory nerve (n. XI) Rhomboid major muscle

II. CLINICAL ANATOMY The suprascapular artery is a branch of the thyrocervical trunk of the subclavian artery. It passes superior to the suprascapular ligament at the suprascapular notch to supply the supraspinatus and infraspinatus muscles. The suprascapular nerve arises from the upper trunk of the brachial plexus and passes inferior to the ligament to innervate the same muscles. The suprascapular nerve is subject to impingement at the notch, whereas the artery would not be compressed because it passes over the ligament.

Copyright © 2016 Wolters Kluwer

Vilensky_06.indd 242

3/17/15 12:29 AM

Upper Limb

6.6

1

2

5 3

4

6

7

Vilensky_06.indd 243

3/17/15 12:29 AM

I. LABELS 1. 2. 3. 4. 5. 6. 7.

6.6

Supraspinatus muscle Infraspinatus muscle Latissimus dorsi muscle Long head of triceps brachii muscle Deltoid muscle Radial nerve Lateral head of triceps brachii muscle

II. QUESTION After an anterior-inferior humeral dislocation that damaged the nerve that passes through the quadrangular space, you would expect: A. Weakness in humeral abduction and no sensory loss B. Weakness in humeral abduction and sensory loss along the lateral aspect of the upper arm C. Weakness in humeral adduction and no sensory loss D. Weakness in humeral adduction and sensory loss along the lateral aspect of the upper arm E. Weakness in humeral flexion and no sensory loss

Vilensky_06.indd 244

Answer is B

Copyright © 2016 Wolters Kluwer

3/17/15 12:29 AM

Upper Limb

6.7

1 2 3 4 5 6

Vilensky_06.indd 245

3/17/15 12:29 AM

I. LABELS

6.7

Trapezius muscle Supraspinatus muscle Rhomboid major muscle Axillary nerve and posterior circumflex humeral artery 5. Teres minor muscle 6. Teres major muscle

1. 2. 3. 4.

II. CLINICAL ANATOMY The quadrangular space is bounded as follows: • • • •

Superiorly: teres minor muscle Inferiorly: teres major muscle Medially: long head of the triceps brachii muscle Laterally: surgical neck of the humerus

The space transmits the axillary nerve and the posterior humeral circumflex vessels. Both the nerve and the vessels can be compressed by mass lesions in the space. Symptoms may include deltoid weakness and sensory loss on the lateral aspect of the upper part of the arm.

Copyright © 2016 Wolters Kluwer

Vilensky_06.indd 246

3/17/15 12:29 AM

Upper Limb

6.8

1 2 3

6

7 4 5 8

Vilensky_06.indd 247

3/17/15 12:29 AM

I. LABELS

6.8

Spine of scapula Infraspinatus muscle Teres minor muscle Teres major muscle Long head of triceps brachii muscle Deltoid muscle (cut and reflected) Upper lateral cutaneous nerve of arm (branch of axillary nerve) 8. Radial nerve

1. 2. 3. 4. 5. 6. 7.

II. QUESTION A 36-year-old man falls from his bicycle and sustains a midhumeral fracture. Upon examination, he shows “wrist drop.” Which of the following nerves was most likely injured in the fall? Median Radial Musculocutaneous Ulnar Axillary

Copyright © 2016 Wolters Kluwer

Vilensky_06.indd 248

Answer is B

A. B. C. D. E.

3/17/15 12:29 AM

Upper Limb

6.9

4 5

6 1

7

2

3

Vilensky_06.indd 249

3/17/15 12:29 AM

I. LABELS 1. 2. 3. 4. 5. 6. 7.

6.9

Short head of biceps brachii muscle Long head of biceps brachii muscle Medial cutaneous nerve of forearm Subclavius muscle Thoracoacromial artery Pectoral branch of thoracoacromial artery Pectoralis minor muscle

II. CLINICAL ANATOMY The pectoralis minor is normally thought of as acting from the ribs to depress and protract the scapula. However, it is also an effective accessory respiratory muscle in patients with dyspnea. In this case, the muscle acts from its attachment at the coracoid process to raise the ribs during inspiration. In order for this to be effective, the patient has to have the scapula braced so that the patient is typically seated with hands or elbows on a support such a table.

Copyright © 2016 Wolters Kluwer

Vilensky_06.indd 250

3/17/15 12:29 AM

Upper Limb 1

6.10

5 6

2

7

3 4

8

Vilensky_06.indd 251

3/17/15 12:29 AM

I. LABELS 1. 2. 3. 4. 5. 6. 7. 8.

6.10

Insertion of pectoralis major muscle Median nerve Deep axillary lymph node Serratus anterior muscle Short head of biceps brachii muscle Cephalic vein Axillary vein Anterior cutaneous branches of intercostal nerves

II. QUESTION After carrying a heavy log on his right shoulder for many miles, a 33-year-old male is complaining that he is unable to fully elevate his upper limb. You also notice that when he is facing a wall and presses against the wall with his hands, his right scapula protrudes posteriorly (winging of scapula). You suspect injury to which of the following nerves? Dorsal scapular Musculocutaneous Long thoracic Thoracodorsal Greater occipital

Copyright © 2016 Wolters Kluwer

Vilensky_06.indd 252

Answer is C

A. B. C. D. E.

3/17/15 12:29 AM

Upper Limb

6.11 5 6 7

1

8

2 3

4

Vilensky_06.indd 253

3/17/15 12:29 AM

I. LABELS 1. 2. 3. 4. 5. 6. 7. 8.

6.11

Posterior cord of brachial plexus Lateral root of median nerve Musculocutaneous nerve Ulnar nerve Anterior scalene muscle Transverse cervical artery Suprascapular artery Subclavian artery

II. CLINICAL ANATOMY Thoracic outlet syndrome (TOS) results from compression of the neurovascular bundle that passes over the first rib between the anterior scalene and middle scalene muscles (the lower trunk of the brachial plexus and the subclavian artery are often involved). The signs and symptoms of TOS are variable, with the patient often showing neurogenic signs such as numbness, pain, and paresthesia in the upper limb. The fingers may also feel cold due to the arterial compression and subsequent vascular compromise.

Copyright © 2016 Wolters Kluwer

Vilensky_06.indd 254

3/17/15 12:29 AM

Upper Limb

6.12

1 5 2

6

3 7 4

Vilensky_06.indd 255

3/17/15 12:29 AM

I. LABELS 1. 2. 3. 4. 5. 6. 7.

6.12

Long head of biceps brachii muscle Medial cutaneous nerve of forearm Ulnar nerve Lateral cutaneous nerve of forearm Serratus anterior muscle Thoracodorsal nerve and artery Latissimus dorsi muscle

II. QUESTION A patient comes to the ED with a severe knife wound to his arm that severs the biceps brachii and the underlying nerve. Which of the following is the most likely sequelae? A. Weakness in elbow extension B. Reduced sensation on the anterior medial aspect of the arm C. Reduced sensation on the anterior lateral aspect of the arm D. Reduced sensation over the olecranon E. Reduced sensation over the lateral aspect of the forearm

Vilensky_06.indd 256

Answer is E

Copyright © 2016 Wolters Kluwer

3/17/15 12:29 AM

Upper Limb

6.13

1 2

3

4 6 5

Vilensky_06.indd 257

3/17/15 12:29 AM

I. LABELS 1. 2. 3. 4. 5. 6.

6.13

Subscapularis muscle Deltoid muscle Biceps brachii muscle Brachialis muscle Bicipital aponeurosis Brachial artery

II. QUESTION A 12-year-old female child was brought to the ED with a fractured olecranon process. Based on the location of this injury, you are most concerned about damage to which of the following nerves? Median Ulnar Musculocutaneous Radial Anterior interosseous

Copyright © 2016 Wolters Kluwer

Vilensky_06.indd 258

Answer is B

A. B. C. D. E.

3/17/15 12:29 AM

Upper Limb

6.14

1 2 3

4 5

Vilensky_06.indd 259

3/17/15 12:29 AM

6.14

I. LABELS

1. Pectoralis major muscle 2. Long head of biceps brachii in intertubercular sulcus 3. Head of humerus 4. Deltoid muscle 5. Infraspinatus muscle

II. QUESTION Following an injury to the infraspinatus muscle, you would expect to find weakness in which of the following movements? Lateral rotation of humerus Medial rotation of humerus Abduction of humerus Protraction of the scapula Downward rotation of scapula

Copyright © 2016 Wolters Kluwer

Vilensky_06.indd 260

Answer is A

A. B. C. D. E.

3/17/15 12:29 AM

Upper Limb

6.15 3 4

1

5 6

2

Vilensky_06.indd 261

3/17/15 12:29 AM

I. LABELS 1. 2. 3. 4. 5. 6.

6.15

Radial nerve Triceps brachii muscle Cephalic vein Biceps brachii muscle Brachial artery Shaft of humerus

II. CLINICAL ANATOMY The triceps brachii is innervated by the radial nerve. Although the radial nerve travels from the axilla to the cubital fossa, a fracture of the humerus may not affect the action of the triceps because the branches to this muscle arise from the most proximal aspect of the nerve. At the cubital fossa, the radial nerve divides into superficial and deep branches. The superficial branch is a purely sensory nerve, whereas the deep branch is primarily motor to the extensor muscles of the wrist and hand.

Copyright © 2016 Wolters Kluwer

Vilensky_06.indd 262

3/17/15 12:29 AM

6.16

Upper Limb 1 2

3

Vilensky_06.indd 263

4 5 6

7

3/17/15 12:29 AM

I. LABELS 1. 2. 3. 4. 5. 6. 7.

6.16

Articular circumference of radius Radial tuberosity Radial styloid process Trochlear notch Coronoid process Ulnar tuberosity Styloid process of ulna

II. QUESTION A 63-year-old male patient is having difficulty pronating and supinating his forearm and hand. The most proximal joint involved in the process of pronation and supination is which type of joint? Condyloid Ball and socket Pivot Hinge Gliding

Copyright © 2016 Wolters Kluwer

Vilensky_06.indd 264

Answer is C

A. B. C. D. E.

3/17/15 12:29 AM

6.17

Upper Limb 1 5

2

6

3 7

4

Vilensky_06.indd 265

8

3/17/15 12:29 AM

I. LABELS 1. 2. 3. 4. 5. 6. 7. 8.

6.17

Median nerve Brachioradialis muscle Tendon of flexor carpi radialis muscle Palmar aponeurosis Bicipital aponeurosis Palmaris longus muscle Tendon of flexor carpi ulnaris muscle Abductor digiti minimi muscle

II. QUESTION A plastic surgeon needs to use an autologous tendon in a patient who has undergone an extensive neck dissection due to cancer. Which of the following tendons of the forearm is sometimes absent and thus, when present, can be harvested for this procedure with little to no functional impairment? Pronator teres Palmaris longus Anconeus Extensor carpi radialis longus Flexor carpi radialis

Copyright © 2016 Wolters Kluwer

Vilensky_06.indd 266

Answer is B

A. B. C. D. E.

3/17/15 12:29 AM

Upper Limb

6.18

4

5

6 1 7 2 3

Vilensky_06.indd 267

3/17/15 12:29 AM

I. LABELS 1. 2. 3. 4. 5. 6. 7.

6.18

Radial recurrent artery Radial artery Brachioradialis muscle Inferior ulnar collateral artery Brachial artery Pronator teres Ulnar artery

II. CLINICAL ANATOMY There is a rich anastomosis of vessels around the elbow joint, which permits blood flow to the forearm and hand regardless of the position of the elbow joint. Recurrent branches, in some cases double, originate from radial, ulnar, and interosseous arteries and travel proximally both anterior and posterior to the elbow joint. These vessels anastomose with collateral branches of the brachial and deep brachial arteries.

Copyright © 2016 Wolters Kluwer

Vilensky_06.indd 268

3/17/15 12:29 AM

Upper Limb

6.19

1

2

5 3 4

6

7

Vilensky_06.indd 269

3/17/15 12:29 AM

6.19

I. LABELS 1. Biceps brachii muscle 2. Lateral cutaneous nerve of forearm (terminal branch of musculocutaneous nerve) 3. Superficial branch of radial nerve 4. Radial recurrent artery 5. Brachialis muscle 6. Median nerve 7. Pronator teres muscle

II. QUESTION A 27-year-old woman sustained a severe wound in her proximal lateral forearm. In order to determine whether the patient has an injury to the superficial branch of her radial nerve, you would: Assess the function of the pronator teres Assess the function of the flexor pollicis longus Assess the function of the flexor carpi radialis Test for sensation at the wound site Test for sensation on dorsal lateral surface of the hand

Copyright © 2016 Wolters Kluwer

Vilensky_06.indd 270

Answer is E

A. B. C. D. E.

3/17/15 12:29 AM

6.20

Upper Limb 3

4 5 6 1

2

7 8

Vilensky_06.indd 271

3/17/15 12:29 AM

I. LABELS 1. 2. 3. 4. 5. 6. 7. 8.

6.20

Tendon of flexor carpi radialis muscle Flexor pollicis brevis muscle Median nerve Flexor digitorum superficialis muscle Ulnar nerve Tendon of palmaris longus muscle Superficial palmar arch Common palmar digital branch of ulnar nerve

II. QUESTION A 64-year-old man cut his hand resulting in severe bleeding from his superficial palmar arch. This arch is primarily supplied with blood from the: Radial artery Superficial branch of the ulnar artery Deep branch of the ulnar artery Median artery Anterior interosseous artery

Copyright © 2016 Wolters Kluwer

Vilensky_06.indd 272

Answer is B

A. B. C. D. E.

3/17/15 12:29 AM

6.21

Upper Limb 1 5

6 2 3 7

4

Vilensky_06.indd 273

8

3/17/15 12:29 AM

I. LABELS 1. 2. 3. 4.

5. 6. 7. 8.

6.21

Brachialis muscle Tendon of abductor pollicis longus muscle Tendon of flexor pollicis longus muscle Tendons of flexor digitorum profundus muscle (having passed through the divided tendons of the flexor digitorum superficialis muscle) Pronator teres muscle Tendon of flexor carpi ulnaris muscle Lumbrical muscles Tendons of flexor digitorum superficialis muscle (cut)

II. CLINICAL ANATOMY The four lumbrical muscles of the hand arise from the radial side of the flexor digitorum profundus tendons. They insert primarily into the extensor expansion tendons on the dorsal side of digits 2 to 5. They act primarily to extend the interphalangeal joints, although they also can flex the metatarsophalangeal joints. The two medial lumbrical muscles are innervated by the ulnar nerve, whereas the two lateral muscles are innervated by the median nerve. Thus, an injury to the median nerve would paralyze the lumbricals to digits 2 and 3, whereas injury to the ulnar nerve would affect the lumbricals to digits 4 and 5.

Copyright © 2016 Wolters Kluwer

Vilensky_06.indd 274

3/17/15 12:29 AM

Upper Limb

6.22

3 1

2

4

5 6 7

Vilensky_06.indd 275

3/17/15 12:29 AM

I. LABELS 1. 2. 3. 4. 5. 6. 7.

6.22

Brachioradialis muscle Radial recurrent artery Median nerve Anterior interosseous artery Ulnar artery Ulnar nerve Flexor digitorum profundus muscle

II. QUESTION While performing a deep surgical exploration of the proximal forearm for cancer, the surgeon comes across an artery located on the anterior surface of the interosseous membrane. This artery is a direct branch of the: Common interosseous artery Radial artery Radial recurrent artery Brachial artery Median artery

Copyright © 2016 Wolters Kluwer

Vilensky_06.indd 276

Answer is A

A. B. C. D. E.

3/17/15 12:29 AM

6.23

Upper Limb 1 2

3

4 5

Vilensky_06.indd 277

6

3/17/15 12:29 AM

6.23

I. LABELS 1. 2. 3. 4. 5. 6.

Brachioradialis muscle Superficial branch of radial nerve Flexor pollicis longus muscle Flexor retinaculum Median nerve Superficial branch of ulnar nerve

II. QUESTION Edema in which of the following structures could not cause compression of the median nerve, and thus carpal tunnel syndrome, because it does not pass deep to the flexor retinaculum? flexor digitorum superficialis muscle flexor digitorum profundus muscle flexor digitorum superficialis muscle flexor carpi radialis muscle flexor pollicis longus muscle

Copyright © 2016 Wolters Kluwer

Vilensky_06.indd 278

Answer is D

A. Tendon of to digit 5 B. Tendon of to digit 5 C. Tendon of to digit 2 D. Tendon of E. Tendon of

3/17/15 12:29 AM

6.24

Upper Limb 1

2

5

6

3

7

4

Vilensky_06.indd 279

3/17/15 12:29 AM

I. LABELS 1. 2. 3. 4. 5. 6. 7.

6.24

Biceps brachii muscle Pronator teres muscle Opponens pollicis muscle Adductor pollicis muscle Interosseous membrane Pronator quadratus muscle Flexor digiti minimi brevis muscle

II. QUESTION In a patient with a deep knife wound at his thenar eminence, the physician notes that the opponens pollicis is paralyzed. He is able to tell this muscle from the other thenar eminence muscles because: A. It is the only thenar muscle to originate from the trapezium. B. It is the only thenar muscle innervated by the deep branch of the ulnar nerve. C. It is the only thenar muscle that inserts onto the first metacarpal bone. D. It is the only thenar muscle innervated by the recurrent branch of the median nerve. E. It is the only muscle that acts during thumb opposition.

Vilensky_06.indd 280

Answer is C

Copyright © 2016 Wolters Kluwer

3/17/15 12:29 AM

6.25

Upper Limb 4

5 1 6

7 2 3

Vilensky_06.indd 281

3/17/15 12:29 AM

I. LABELS 1. 2. 3. 4. 5. 6. 7.

6.25

Radial artery Median nerve Superficial palmar arch Inferior ulnar collateral artery Ulnar recurrent artery Ulnar nerve Ulnar artery

II. CLINICAL ANATOMY The ulnar canal or ulnar tunnel, also called Guyon’s canal, is a space at the wrist between the pisiform bone and the hamate bone. It is traversed by the ulnar artery and the ulnar nerve. Guyon’s canal syndrome, sometimes called handlebar palsy, results from entrapment of the ulnar nerve in the canal. Symptoms usually begin with a feeling of pins and needles in the fourth and fifth digits; these symptoms may progress to a loss of sensation and/or paresis/paralysis of many of the intrinsic muscles of the hand.

Copyright © 2016 Wolters Kluwer

Vilensky_06.indd 282

3/17/15 12:29 AM

6.26

Upper Limb 1

5

6 2

7

3 8

4

Vilensky_06.indd 283

9

3/17/15 12:29 AM

6.26

I. LABELS 1. 2. 3. 4. 5. 6. 7. 8. 9.

Anconeus muscle Extensor carpi ulnaris muscle Extensor retinaculum Tendons of extensor digiti minimi muscle Extensor carpi radialis longus muscle Extensor carpi radialis brevis muscle Abductor pollicis longus muscle Tendon of extensor pollicis longus muscle First dorsal interosseous muscle

II. QUESTION The “anatomical snuffbox” is a triangular depression on the posterolateral side of the wrist bounded by the tendons of extensor pollicis longus medially and extensor pollicis brevis and abductor pollicis longus laterally. Pain in the anatomical snuffbox after a fall on an outstretched hand may indicate a fracture of which bone? Distal radius First metacarpal Scaphoid Lunate Hamate

Copyright © 2016 Wolters Kluwer

Vilensky_06.indd 284

Answer is C

A. B. C. D. E.

3/17/15 12:29 AM

6.27

Upper Limb 3

1 4

5 6 2

Vilensky_06.indd 285

7

3/17/15 12:29 AM

I. LABELS 1. 2. 3. 4. 5. 6. 7.

6.27

Extensor carpi ulnaris muscle Fourth dorsal interosseous muscle Brachioradialis muscle Extensor carpi radialis brevis muscle Extensor indicis muscle Tendon of extensor carpi radialis longus muscle First dorsal interosseous muscle

II. QUESTION Which of the following movements can be used to test the function of the first dorsal interosseous muscle? Opposition of the thumb Abduction of the thumb Abduction of the index finger Adduction of the index finger Flexion of the proximal interphalangeal joint of the index finger

Copyright © 2016 Wolters Kluwer

Vilensky_06.indd 286

Answer is C

A. B. C. D. E.

3/17/15 12:29 AM

Upper Limb

1

6.28

4 5

2 3

Vilensky_06.indd 287

6

3/17/15 12:29 AM

I. LABELS 1. 2. 3. 4. 5. 6.

6.28

Scaphoid Trapezium Capitate Lunate Pisiform Hamulus or hook of hamate

II. CLINICAL ANATOMY A fractured scaphoid is often difficult to detect using radiography immediately after the fracture, although it can typically be easily seen using MRI. Furthermore, a transverse fracture of the scaphoid bone at its “waist” may result in avascular necrosis of the proximal segment because the blood supply to the scaphoid enters the bone distally.

Copyright © 2016 Wolters Kluwer

Vilensky_06.indd 288

3/17/15 12:29 AM

Upper Limb

6.29

1

3 2

4

5

6

Vilensky_06.indd 289

3/17/15 12:29 AM

I. LABELS 1. 2. 3. 4. 5. 6.

6.29

Pronator quadratus muscle Tendon of flexor pollicis longus muscle Dorsal interosseous muscle Palmar interosseous muscle Tendons of flexor digitorum superficialis muscle Tendons of flexor digitorum profundus muscle

II. CLINICAL ANATOMY Injury to the lower parts of the brachial plexus typically results in a “claw hand” deformity because most of the intrinsic hand muscles are innervated by the C8 and T1 spinal segments. A claw hand is produced by a combination of hyperextension at the metacarpophalangeal (MP) joints and flexion at the interphalangeal (IP) joints due to paralysis of the lumbrical and interossei muscles, which are necessary for producing flexion at the MP joints and extension at the IP joints.

Copyright © 2016 Wolters Kluwer

Vilensky_06.indd 290

3/17/15 12:29 AM

Upper Limb

6.30

1 2

4 5

3

Vilensky_06.indd 291

6

3/17/15 12:29 AM

6.30

I. LABELS 1. 2. 3. 4. 5. 6.

Opponens pollicis muscle Oblique head of adductor pollicis muscle Transverse head of adductor pollicis muscle Opponens digiti minimi muscle Flexor digiti minimi brevis muscle Second palmar interosseous muscle

II. QUESTION An elderly woman has been having difficulty holding objects between her thumb and index finger. Her physician suspects that her adductor pollicis muscle may not be functioning properly. Which of the following statements best described the innervation of adductor pollicis muscle? A. Both heads are innervated by the deep branch of the ulnar nerve. B. Both heads are innervated by the median nerve. C. Both heads are innervated by the radial nerve. D. The transverse head is innervated by the deep branch of the ulnar nerve and the oblique head by the median nerve. E. The transverse head is innervated by the median nerve and the oblique head by the deep branch of the ulnar nerve.

Vilensky_06.indd 292

Answer is A

Copyright © 2016 Wolters Kluwer

3/17/15 12:30 AM

Upper Limb

6.31

3

1

4 2

5 6

Vilensky_06.indd 293

3/17/15 12:30 AM

6.31

I. LABELS Superficial palmar branch of radial artery Flexor pollicis brevis muscle Flexor carpi ulnaris tendon Abductor digiti minimi muscle Common palmar digital nerve (branch of ulnar nerve) 6. Common palmar digital artery

1. 2. 3. 4. 5.

II. QUESTION A 35-year-old woman cut the dorsal side of her fifth digit (pinky finger) with a paper cutter. The cut was deep enough that she required stitches. Which nerve would have to be anesthetized to allow for painless suturing of her wound? Superficial radial nerve Deep radial nerve Dorsal cutaneous branch of ulnar nerve Recurrent branch of median nerve Palmar cutaneous branch of median nerve

Copyright © 2016 Wolters Kluwer

Vilensky_06.indd 294

Answer is C

A. B. C. D. E.

3/17/15 12:30 AM

Upper Limb 1

6.32 5

2

3

6

4

7

Vilensky_06.indd 295

3/17/15 12:30 AM

I. LABELS

6.32

Radial artery Tendon of flexor carpi radialis muscle Abductor pollicis brevis muscle Oblique and transverse heads of adductor pollicis muscle 5. Pronator quadratus muscle 6. Abductor digiti minimi muscle 7. Palmar digital artery of the fifth digit

1. 2. 3. 4.

II. QUESTION A 27-year-old male sustained a knife wound to his forearm that damaged his flexor carpi radialis muscle. He would likely experience weakness of which of the following movements? Wrist flexion and extension Wrist flexion and abduction Wrist flexion and adduction Wrist and thumb flexion Thumb flexion and opposition

Copyright © 2016 Wolters Kluwer

Vilensky_06.indd 296

Answer is B

A. B. C. D. E.

3/17/15 12:30 AM

Upper Limb 1

6.33 4

2 3 5

6

Vilensky_06.indd 297

3/17/15 12:30 AM

I. LABELS 1. 2. 3. 4. 5. 6.

6.33

Radial artery Deep palmar arch Princeps pollicis artery Ulnar artery Superficial palmar arch Proper palmar digital arteries

II. CLINICAL ANATOMY The princeps pollicis artery (principal artery of the thumb) branches off of the radial artery as it enters the deep part of the palm; the princeps pollicis artery passes between the first dorsal interosseous muscle and the oblique head of the adductor pollicis. It then accompanies the first metacarpal bone (medial side) to the base of the proximal phalanx, where it can be found deep to the tendon of the flexor pollicis longus muscle; it then divides into two branches to supply the digit. One of the reasons that a clinician palpates the pulse of a patient using the tips of the index and middle fingers is to avoid feeling his or her own pulse of the princeps pollicis artery.

Copyright © 2016 Wolters Kluwer

Vilensky_06.indd 298

3/17/15 12:30 AM

Upper Limb 1

6.34 5

2 3

6

4

Vilensky_06.indd 299

3/17/15 12:30 AM

I. LABELS 1. 2. 3. 4. 5. 6.

6.34

Basilic vein Radial artery Ulnar artery Ulna Median cubital vein Radius

II. CLINICAL ANATOMY Because of its superficial position overlying the cubital fossa, the median cubital vein is commonly used for venipuncture. It crosses the cubital fossa anterior to the bicipital aponeurosis, which helps protect the underlying structures within the cubital fossa, namely the brachial artery and median nerve. These structures may still be injured during venipuncture, however, if the needle is inserted too deeply or too far medially in the cubital fossa.

Copyright © 2016 Wolters Kluwer

Vilensky_06.indd 300

3/17/15 12:30 AM

Upper Limb

6.35

1 3 4 2 5

Vilensky_06.indd 301

3/17/15 12:30 AM

I. LABELS 1. 2. 3. 4. 5.

6.35

Second and third metacarpal bones Thenar muscles Fifth metacarpal bone Hypothenar muscles Tendons of flexor digitorum superficialis and profundus muscles

II. CLINICAL ANATOMY In a T1-weighted MR image such as this one, fat is bright. So in this image, it is actually the marrow of the metacarpal bones that makes them appear bright. The surrounding cortical bone has no signal and therefore appears very dark.

Copyright © 2016 Wolters Kluwer

Vilensky_06.indd 302

3/17/15 12:30 AM

Head and Neck

7.1

1 5 2

3

6 7

8 4

Vilensky_07.indd 303

3/17/15 1:19 AM

I. LABELS 1. 2. 3. 4. 5. 6. 7. 8.

7.1

Coronal suture Sphenoid bone (greater wing) Articular tubercle (temporal bone) Mental foramen Squamosal suture External acoustic meatus Mastoid process Angle of the mandible

II. CLINICAL ANATOMY If a person loses his or her mandibular teeth, the alveolar process of the mandible (the portion that supports the teeth) degenerates. When this happens, the mental foramen often comes to lie on the superior aspect of the body of the mandible so that with each chewing motion, pressure is placed on the nerve that exits from the foramen, the mental nerve. This is quite painful to the person while eating, especially if the person is wearing dentures.

Copyright © 2016 Wolters Kluwer

Vilensky_07.indd 304

3/17/15 1:19 AM

Head and Neck

1 2

7.2

5

3 4

6 7

8

Vilensky_07.indd 305

3/17/15 1:19 AM

I. LABELS 1. 2. 3. 4. 5. 6. 7. 8.

7.2

Supraorbital notch (foramen) Superior orbital fissure Middle nasal concha (turbinate) Inferior nasal concha (turbinate) Frontonasal suture Nasal septum (vomer) Ramus of mandible Mental foramen

II. QUESTION Following severe head trauma, a patient has edema within the superior orbital fissure. Which one of the following nerves would not likely be affected by such swelling? Trochlear Oculomotor Maxillary Frontal Nasociliary

Copyright © 2016 Wolters Kluwer

Vilensky_07.indd 306

Answer is C

A. B. C. D. E.

3/17/15 1:19 AM

Head and Neck 1

7.3

4 5

2 3

Vilensky_07.indd 307

3/17/15 1:19 AM

I. LABELS 1. 2. 3. 4. 5.

7.3

Optic canal Lateral pterygoid plate of pterygoid process Pterygoid hamulus Superior orbital fissure Foramen rotundum

II. CLINICAL ANATOMY As the optic nerve (cranial nerve II) travels within the optic canal, the periosteum of the canal becomes firmly affixed to the dura surrounding the nerve. This can be significant in head trauma because a blow to the head can cause the nerve fibers within the nerve to shear due to this attachment. This can result in “indirect” trauma to the nerve (tearing of fibers) and cause visual abnormalities, including blindness.

Copyright © 2016 Wolters Kluwer

Vilensky_07.indd 308

3/17/15 1:19 AM

Head and Neck

7.4

1 2 3

5 6

4

Vilensky_07.indd 309

3/17/15 1:19 AM

7.4

I. LABELS 1. 2. 3. 4. 5. 6.

Lesser wing of sphenoid bone Sphenoidal sinus Pterygoid canal Pterygoid hamulus Orbital surface of greater wing of sphenoid Foramen rotundum

II. QUESTION Following head trauma, a CT scan reveals a fracture that passes through the foramen rotundum. Which of the following would not be consistent with transection of the nerve that passes through this canal? Loss of salivation from the parotid gland Decreased tearing Decreased nasal secretions Decreased taste from hard palate Decreased taste from soft palate

Copyright © 2016 Wolters Kluwer

Vilensky_07.indd 310

Answer is A

A. B. C. D. E.

3/17/15 1:19 AM

Head and Neck

7.5

3

4 1 5 2

Vilensky_07.indd 311

3/17/15 1:19 AM

I. LABELS 1. 2. 3. 4. 5.

7.5

Occipital condyle Pharyngeal tubercle External occipital protuberance Condylar canal Foramen magnum

II. QUESTION In a patient with a tumor compressing the structures that pass through the foramen magnum, you would expect to see which of the following in addition to neurological signs affecting the torso and limbs? Difficulty speaking Difficulty swallowing Reduced blood supply to brain Decreased hearing acuity Decreased CSF pressure

Copyright © 2016 Wolters Kluwer

Vilensky_07.indd 312

Answer is C

A. B. C. D. E.

3/17/15 1:19 AM

Head and Neck

7.6

1

2 3 4 5

Vilensky_07.indd 313

3/17/15 1:19 AM

I. LABELS 1. 2. 3. 4. 5.

7.6

Squamous part of temporal bone Zygomatic process of temporal bone Mandibular fossa External acoustic meatus Mastoid process

II. CLINICAL ANATOMY The joint between the mandible and the temporal bone (temporomandibular joint [TMJ]) is located at the mandibular fossa. This joint is a bicondylar joint, actually involving the joints of both sides. Within each TMJ, an articular disk separates the joint into upper and lower joint cavities. The upper joint cavity is a sliding joint, allowing protrusion and retrusion of the mandible. The lower joint cavity primarily acts as a hinge joint, but some rotation is permitted. This is associated with the grinding motion of the teeth during chewing. Another example of a bicondylar joint is the knee, although in this case, the two condyles are contained within each joint cavity.

Copyright © 2016 Wolters Kluwer

Vilensky_07.indd 314

3/17/15 1:19 AM

Head and Neck

7.7 4

1 2

5 6

3

Vilensky_07.indd 315

3/17/15 1:19 AM

I. LABELS 1. 2. 3. 4. 5. 6.

7.7

Zygomatic process of frontal bone Anterior ethmoidal foramen (canal) Posterior ethmoidal foramen (canal) Supraorbital notch Orbital plate of frontal bone Ethmoidal notch

II. QUESTION A 28-year-old male boxer was punched in the face, and the bones comprising his zygomatic arch were fractured. Some of the bone fragments were now embedded in the muscle directly deep (medial) to the arch. This muscle is: Temporalis Masseter Zygomaticus major Lateral pterygoid Medial pterygoid

Copyright © 2016 Wolters Kluwer

Vilensky_07.indd 316

Answer is A

A. B. C. D. E.

3/17/15 1:19 AM

Head and Neck

7.8

1

2 3

7

4

8

5 6

9

10

Vilensky_07.indd 317

3/17/15 1:19 AM

I. LABELS 1. 2. 3. 4. 5. 6. 7. 8. 9. 10.

7.8

Frontal crest Cribriform plate of ethmoidal bone Lesser wing of sphenoidal bone Foramen rotundum Foramen spinosum Jugular foramen Anterior clinoid process Sella turcica (hypophysial fossa) Groove for superior petrosal sinus Diploë

II. CLINICAL ANATOMY The olfactory nerve filaments pass from the olfactory mucosa in the upper part of the nasal cavity through the foramina of the cribriform plate to reach the olfactory bulb of the brain. Together, these nerve filaments comprise the second cranial nerve, the olfactory nerve. It is believed that some neurological conditions such as Parkinson’s disease may develop as a result of viruses following these nerves through these foramina to reach the brain.

Copyright © 2016 Wolters Kluwer

Vilensky_07.indd 318

3/17/15 1:19 AM

Head and Neck 1

7.9

4

5 2

3

Vilensky_07.indd 319

3/17/15 1:19 AM

I. LABELS 1. 2. 3. 4. 5.

7.9

Sagittal suture Lambdoid suture External occipital protuberance Parietal foramen Lambda

II. CLINICAL ANATOMY Emissary veins pass through the parietal foramina connecting the veins in the scalp to the superior sagittal sinus. These veins are valveless and can therefore transmit infected thrombi from the scalp to the cranial cavity, leading to encephalitis.

Copyright © 2016 Wolters Kluwer

Vilensky_07.indd 320

3/17/15 1:19 AM

Head and Neck

7.10

1

2

3

4

Vilensky_07.indd 321

5

6

3/17/15 1:19 AM

I. LABELS 1. 2. 3. 4. 5. 6.

7.10

Incisive fossa Palatine process of maxilla Horizontal plate of palatine Choanae Greater palatine foramen Lesser palatine foramen

II. QUESTION A dentist is about to inject an anesthetic solution through the incisive foramen to anesthetize the anterior part of the hard palate. The nerve that passes through this foramen is a branch of the: Lateral nasal nerve Facial nerve Lingual nerve Nasopalatine nerve Glossopharyngeal nerve

Copyright © 2016 Wolters Kluwer

Vilensky_07.indd 322

Answer is D

A. B. C. D. E.

3/17/15 1:19 AM

Head and Neck

7.11 4

5

1 2

6

3

Vilensky_07.indd 323

3/17/15 1:19 AM

I. LABELS 1. 2. 3. 4. 5. 6.

7.11

Orbital plate of ethmoid bone (lamina papyracea) Fossa for lacrimal sac Infraorbital foramen Anterior and posterior ethmoidal foramina Greater wing of sphenoidal bone Inferior orbital fissure

II. CLINICAL ANATOMY The lamina papyracea is the part of the ethmoid bone that makes up the medial wall of the orbit. It is very thin and therefore fractures easily when there is a blow-out fracture of the orbit, as occurs when a person is hit in the orbit by a blunt object such as a softball. This thin-walled border also may be penetrated by infections, enabling them to spread from the ethmoidal sinus to the orbit.

Copyright © 2016 Wolters Kluwer

Vilensky_07.indd 324

3/17/15 1:19 AM

Head and Neck

7.12

1

5

2

6

3

4

7

Vilensky_07.indd 325

3/17/15 1:19 AM

I. LABELS 1. 2. 3. 4. 5. 6. 7.

7.12

Condylar process Articular tubercle Mandibular notch Angle of mandible Zygomatic bone Coronoid process Mental foramen

II. QUESTION A 36-year-old female patient is having difficulty opening her mouth against resistance. You suspect damage to which of the following muscles that attaches to the mandibular condyle? Medial pterygoid Lateral pterygoid Anterior belly of digastric Buccinator Masseter

Copyright © 2016 Wolters Kluwer

Vilensky_07.indd 326

Answer is B

A. B. C. D. E.

3/17/15 1:19 AM

Head and Neck

7.13 5

1 2

3

6 7

4

Vilensky_07.indd 327

3/17/15 1:19 AM

I. LABELS 1. 2. 3. 4. 5. 6. 7.

7.13

Lateral pterygoid muscle Medial pterygoid muscle Posterior belly of digastric muscle Masseter muscle Temporal muscle Parotid duct (cut) Buccinator muscle

II. QUESTION While chewing, a 47-year-old male patient is continually getting food stuck between his teeth and cheek on the left side of his mouth. You suspect that his left buccinator may be paralyzed. This muscle is innervated by which of the following nerves? Lingual Facial Glossopharyngeal Deep temporal Vagus

Copyright © 2016 Wolters Kluwer

Vilensky_07.indd 328

Answer is B

A. B. C. D. E.

3/17/15 1:19 AM

Head and Neck

1

7.14

4

2

5

3

6

Vilensky_07.indd 329

3/17/15 1:19 AM

7.14

I. LABELS 1. 2. 3. 4. 5. 6.

Temporal muscle External acoustic meatus Masseter muscle Zygomatic arch Temporomandibular joint Buccinator muscle

II. QUESTION A patient is suspected to have a facial nerve palsy (Bell’s palsy). Which of the following muscles would not be affected? Buccinator Orbicularis oculi Temporalis Orbicularis oris Platysma

Copyright © 2016 Wolters Kluwer

Vilensky_07.indd 330

Answer is C

A. B. C. D. E.

3/17/15 1:19 AM

Head and Neck

7.15 3

1

2 4

Vilensky_07.indd 331

3/17/15 1:19 AM

I. LABELS 1. 2. 3. 4.

7.15

Lateral pterygoid muscles Medial pterygoid muscle Temporomandibular joint Mandible

II. CLINICAL ANATOMY The temporomandibular joint (TMJ) may be dislocated anteriorly (even by an excessive yawn) such that the mandibular condyle comes to lie anterior to the articular tubercle of the temporal bone. To reposition, the body of the mandible must be pushed inferiorly and then posteriorly.

Copyright © 2016 Wolters Kluwer

Vilensky_07.indd 332

3/17/15 1:19 AM

Head and Neck

7.16

1

2 3

4

5

6

7

Vilensky_07.indd 333

3/17/15 1:19 AM

I. LABELS 1. 2. 3. 4. 5. 6. 7.

7.16

Frontal belly of occipitofrontalis muscle Palpebral part of orbicularis oculi muscle Levator labii superioris muscle Orbicularis oris muscle Zygomaticus major muscle Depressor anguli oris muscle Platysma muscle

II. CLINICAL ANATOMY The depressor anguli oris muscle is innervated by the marginal mandibular branch of the facial nerve. This nerve runs very superficially (immediately deep to the platysma muscle) along the body of the mandible and is easily cut during facial surgery. Although the loss of this muscle would seem trivial, it is not because the depressor anguli oris is very important for making normal facial expressions, including smiling. Facial surgeons typically try to stay superficial to the platysma muscle in this region so that they do not inadvertently transect the nerve.

Copyright © 2016 Wolters Kluwer

Vilensky_07.indd 334

3/17/15 1:19 AM

Head and Neck 1

7.17

5

2 3 6 4

7

8 9

Vilensky_07.indd 335

3/17/15 1:19 AM

I. LABELS 1. 2. 3. 4. 5. 6. 7. 8. 9.

7.17

Temporal fascia Zygomatic arch Styloid process Stylohyoid muscle Orbital part of orbicularis oculi muscle Depressor anguli oris muscle Mylohyoid muscle Sternohyoid muscle Omohyoid muscle

II. CLINICAL ANATOMY Eagle syndrome (styloid process–carotid artery syndrome) is a condition associated with an exceptionally long styloid process (more than 30 mm) that interferes with adjacent structures. The patient typically presents with unilateral sore throat, swallowing difficulties, tinnitus, facial and neck pain, and earache. The elongated styloid process can also cause compression of the internal carotid artery, particularly when the head is turned.

Copyright © 2016 Wolters Kluwer

Vilensky_07.indd 336

3/17/15 1:19 AM

Head and Neck

7.18

1

2

6 7 3

4

5

Vilensky_07.indd 337

8

3/17/15 1:19 AM

I. LABELS 1. 2. 3. 4. 5. 6. 7. 8.

7.18

Galea aponeurotica Superficial temporal artery External carotid artery Great auricular nerve Supraclavicular nerves Maxillary artery Buccal nerve (of trigeminal nerve) Transverse cervical nerve

II. CLINICAL ANATOMY The supraclavicular nerves provide cutaneous innervation to the neck and shoulder and contain nerve elements from the third and fourth cervical nerves. The phrenic nerve, which innervates the diaphragm, also contains nerve elements from the third and fourth cervical nerves. This explains why pain from the diaphragm and surrounding area may be referred to the shoulder.

Copyright © 2016 Wolters Kluwer

Vilensky_07.indd 338

3/17/15 1:19 AM

Head and Neck

7.19

1 4 2 3

5 6

7

Vilensky_07.indd 339

3/17/15 1:19 AM

7.19

I. LABELS Olfactory tract Oculomotor nerve (n. III) Abducent nerve (n. VI) Optic nerve (n. II) Trigeminal nerve (n. V) Facial nerve (n. VII), nervus intermedius, and vestibulocochlear nerve (n. VIII) 7. Straight sinus

1. 2. 3. 4. 5. 6.

II. QUESTION A 59-year-old female diabetic patient has been experiencing double vision and has great difficulty abducting her left eye without elevating or raising it at the same time. The patient likely has damage to which of the following nerves on her left side? Oculomotor Frontal Abducent Trochlear Lacrimal

Copyright © 2016 Wolters Kluwer

Vilensky_07.indd 340

Answer is C

A. B. C. D. E.

3/17/15 1:19 AM

Head and Neck

7.20

7 1

2 3 4 5

8 9

10

6

Vilensky_07.indd 341

3/17/15 1:19 AM

I. LABELS 1. 2. 3. 4. 5. 6. 7. 8. 9. 10.

7.20

Auriculotemporal nerve Occipital artery Parotid gland Splenius capitis muscle Trapezius muscle External jugular vein Supraorbital nerves Zygomaticus major muscle Zygomatic and buccal branches of facial nerve Platysma muscle

II. CLINICAL ANATOMY The secretory cells within the parotid gland are innervated by the glossopharyngeal nerve (IX). However, the postganglionic parasympathetic fibers to this gland synapse in the otic ganglion in the infratemporal fossa and then travel with the auriculotemporal nerve to reach the gland. This is one example in which autonomic fibers in the head “hitch a ride” with branches of the trigeminal nerve to reach their destination.

Copyright © 2016 Wolters Kluwer

Vilensky_07.indd 342

3/17/15 1:19 AM

Head and Neck

7.21

1 2 3

7

4

5

6

8 9

Vilensky_07.indd 343

3/17/15 1:19 AM

I. LABELS 1. 2. 3. 4. 5. 6. 7. 8. 9.

7.21

Facial nerve (n. VII) Great auricular nerve Parotid plexus Sternocleidomastoid muscle Cervical branch of facial nerve Transverse cervical nerve Facial artery Anterior belly of digastric muscle Omohyoid muscle

II. QUESTION Torticollis is associated with spasticity in which of the following muscles? Sternocleidomastoid muscle Omohyoid muscle Digastric muscle (both bellies) Zygomaticus major muscle Orbicularis oris muscle

Copyright © 2016 Wolters Kluwer

Vilensky_07.indd 344

Answer is A

A. B. C. D. E.

3/17/15 1:19 AM

Head and Neck

7.22

1 2

6 7

3

4 5

Vilensky_07.indd 345

8

3/17/15 1:19 AM

I. LABELS 1. 2. 3. 4. 5. 6. 7. 8.

7.22

Superficial temporal artery Auriculotemporal nerve Masseteric artery and nerve External carotid artery Sternocleidomastoid muscle Orbicularis oculi muscle Angular artery Facial artery

II. CLINICAL ANATOMY The orbicularis oculi muscle closes the eye and is innervated by the facial nerve. In Bell’s palsy, this nerve is temporarily weakened or paralyzed, and the patient may have to wear an eye patch to protect the eye.

Copyright © 2016 Wolters Kluwer

Vilensky_07.indd 346

3/17/15 1:19 AM

Head and Neck

7.23

1 2 5 6 3 7 4

Vilensky_07.indd 347

8

3/17/15 1:19 AM

I. LABELS 1. 2. 3. 4. 5. 6. 7. 8.

7.23

Frontal branch of superficial temporal artery Auriculotemporal nerve Lateral pterygoid muscle Posterior belly of digastric muscle Deep temporal artery Posterior superior alveolar arteries Lingual nerve Inferior alveolar artery and nerve (mandibular canal opened)

II. CLINICAL ANATOMY The inferior alveolar and lingual nerves are typically anesthetized when dentists work on lower teeth. Occasionally, the injection procedure damages the nerves, producing anesthesia or paresthesia of the teeth, tongue, mouth, and chin on that side that may be prolonged and very annoying to the patient. The chin is numb because the mental nerve is a terminal branch of the inferior alveolar nerve.

Copyright © 2016 Wolters Kluwer

Vilensky_07.indd 348

3/17/15 1:19 AM

Head and Neck

7.24

1

2 3

7 8

4 5 6

Vilensky_07.indd 349

3/17/15 1:19 AM

I. LABELS

7.24

1. Superficial temporal artery and auriculotemporal nerve 2. Maxillary artery 3. Buccal nerve (of trigeminal nerve) 4. Submandibular ganglion 5. Mylohyoid nerve 6. Anterior belly of digastric muscle 7. Great auricular nerve 8. Lesser occipital nerve

II. QUESTION An infection of the submandibular ganglion would affect: Taste from the anterior two-thirds of the tongue Taste from the posterior one-third of the tongue Movements of the tongue Blood supply to the tongue Salivation from the sublingual gland

Copyright © 2016 Wolters Kluwer

Vilensky_07.indd 350

Answer is E

A. B. C. D. E.

3/17/15 1:19 AM

Head and Neck

7.25

5 6 7 1

8

2 3

4

Vilensky_07.indd 351

3/17/15 1:19 AM

I. LABELS 1. 2. 3. 4. 5. 6. 7. 8.

7.25

Superior oblique muscle Superior rectus muscle Levator palpebrae superioris muscle Common annular tendon (annulus tendineus) Trochlea Tendon of superior oblique muscle Medial rectus muscle Optic nerve (extracranial part)

II. QUESTION A 62-year-old male patient has lost the function of his superior oblique muscle. You would expect his diplopia would be most pronounced when he looks: Up Down Medially Laterally Up and medially

Copyright © 2016 Wolters Kluwer

Vilensky_07.indd 352

Answer is B

A. B. C. D. E.

3/17/15 1:19 AM

Head and Neck

1

7.26

5 6

2 3

7

4

Vilensky_07.indd 353

3/17/15 1:19 AM

I. LABELS 1. 2. 3. 4. 5. 6. 7.

7.26

Lacrimal gland Lacrimal nerve Frontal nerve Oculomotor nerve (n. III) Olfactory bulb Levator palpebrae superioris muscle Trochlear nerve (n. IV)

II. QUESTION Your aged patient is complaining of a condition that typically accompanies old age as a result of changes in the lacrimal gland. The patient is likely complaining of: Dry eyes Poor night vision Poor reading vision Cloudiness in vision Glare

Copyright © 2016 Wolters Kluwer

Vilensky_07.indd 354

Answer is A

A. B. C. D. E.

3/17/15 1:19 AM

7.27

Head and Neck

4 5 1 2

6

3

Vilensky_07.indd 355

3/17/15 1:19 AM

I. LABELS 1. 2. 3. 4. 5. 6.

7.27

Lacrimal nerve and artery Lateral rectus muscle Nasociliary nerve Long ciliary nerve Anterior ethmoidal artery and nerve Ophthalmic artery

II. QUESTION Disruption of sympathetic innervation to the orbit would result in which of the following? Pupillary dilation Loss of near accommodation Ptosis (droopy eyelid) Inability to produce tears Bulging eyeball (exophthalmos)

Copyright © 2016 Wolters Kluwer

Vilensky_07.indd 356

Answer is C

A. B. C. D. E.

3/17/15 1:19 AM

Head and Neck

7.28

3

1

4

2

5

Vilensky_07.indd 357

3/17/15 1:19 AM

I. LABELS 1. 2. 3. 4. 5.

7.28

Short ciliary nerves Ciliary ganglion Superior oblique muscle Inferior branch of oculomotor nerve Trochlear nerve (intracranial part)

II. QUESTION A 23-year-old female patient with an infection of her ciliary ganglion is likely to complain of: Dry eyes Reading difficulties Inability to completely abduct her eye Night vision difficulties Loss of sensation from cornea

Copyright © 2016 Wolters Kluwer

Vilensky_07.indd 358

Answer is B

A. B. C. D. E.

3/17/15 1:19 AM

Head and Neck

7.29

1 5 2

6

3 7 4

Vilensky_07.indd 359

3/17/15 1:19 AM

I. LABELS 1. 2. 3. 4. 5. 6. 7.

7.29

Parotid gland Lesser occipital nerve Great auricular nerve Transverse cervical nerve Facial artery Submandibular gland Thyroid cartilage

II. QUESTION The submandibular gland receives its parasympathetic innervation via the: Otic ganglion Facial nerve Glossopharyngeal nerve Maxillary nerve Vagus nerve

Copyright © 2016 Wolters Kluwer

Vilensky_07.indd 360

Answer is B

A. B. C. D. E.

3/17/15 1:19 AM

Head and Neck 1

7.30 4

2 5 3

6

Vilensky_07.indd 361

3/17/15 1:19 AM

I. LABELS 1. 2. 3. 4. 5. 6.

7.30

External carotid artery Ansa cervicalis Superior belly omohyoid muscle Superficial cervical lymph node Middle scalene muscle Subclavius muscle

II. CLINICAL ANATOMY Three hundred of the estimated 800 lymph nodes in the body are situated in the neck. Lymph nodes in the head and neck are arranged in two horizontal rings and two vertical chains on either side of the neck. The outer (superficial) ring consists of the occipital, preauricular (parotid), submandibular, and submental nodes; the inner (deep) ring is formed by clumps of mucosa-associated lymphoid tissue (MALT) located primarily in the nasopharynx and oropharynx (Waldeyer’s ring). The vertical chain consists of superior and inferior groups of deep cervical nodes related to the carotid sheath (jugular groups). All lymph vessels of the head and neck drain into the deep cervical nodes, either directly from the tissues or indirectly via nodes in outlying groups. Lymph is returned to the systemic venous circulation via either the right lymphatic duct or the thoracic duct.

Copyright © 2016 Wolters Kluwer

Vilensky_07.indd 362

3/17/15 1:19 AM

Head and Neck 1 2

7.31

5 6

3 4

7 8

Vilensky_07.indd 363

3/17/15 1:19 AM

I. LABELS 1. 2. 3. 4. 5. 6. 7. 8.

7.31

Mylohyoid nerve Anterior belly of digastric muscle Superior thyroid artery Omohyoid muscle Vagus nerve (n. X) Internal carotid artery Common carotid artery Ansa cervicalis

II. CLINICAL ANATOMY Typically, the common and internal carotid arteries have no branches in the neck. The external carotid has many branches, with the superior thyroid artery typically being the first (most inferior). This artery is ligated during surgical removal of the thyroid gland. The inferior thyroid artery also supplies the gland, and it arises from the thyrocervical trunk of the subclavian artery.

Copyright © 2016 Wolters Kluwer

Vilensky_07.indd 364

3/17/15 1:19 AM

Head and Neck

7.32

1

3 2

4 5 6

Vilensky_07.indd 365

3/17/15 1:19 AM

I. LABELS 1. 2. 3. 4. 5. 6.

7.32

Facial artery Ansa cervicalis Middle scalene muscle Anterior scalene muscle Inferior thyroid artery Dorsal scapular artery

II. QUESTION When the brachial plexus needs to be injected with an anesthetic so that surgery may be done on the upper limb, one approach is to locate the anterior scalene muscle in the neck. This muscle is located: A. Anterior to the upper trunk but posterior to the lower and middle trunks of the brachial plexus B. Posterior to the lower but anterior to the middle and superior trunks C. Lateral to all the trunks D. Anterior to all of the trunks E. Posterior to all of the trunks

Vilensky_07.indd 366

Answer is D

Copyright © 2016 Wolters Kluwer

3/17/15 1:20 AM

Head and Neck

7.33 3

1

4

5

2

Vilensky_07.indd 367

6

3/17/15 1:20 AM

7.33

I. LABELS 1. 2. 3. 4. 5. 6.

Sphenoidal sinus Uvula Olfactory nerves Nasal septum Incisive canal with nasopalatine nerve Hard plate

II. QUESTION A 45-year-old female patient has severe bleeding from her nasal septum. Pressure on which of the following arteries would have the greatest effect on reducing the arterial blood flowing from her left nostril? Left Left Left Left Left

external carotid internal carotid ophthalmic ascending pharyngeal posterior ethmoidal

Copyright © 2016 Wolters Kluwer

Vilensky_07.indd 368

Answer is A

A. B. C. D. E.

3/17/15 1:20 AM

Head and Neck

7.34

1 2 3 5 4

Vilensky_07.indd 369

6

3/17/15 1:20 AM

I. LABELS

7.34

1. Superior rectus and levator palpebrae superioris muscles 2. Superior oblique muscle 3. Inferior rectus and inferior oblique muscles 4. Nasal septum 5. Maxillary sinus 6. Inferior nasal concha

II. CLINICAL ANATOMY In a blow-out fracture that damages the floor of the orbit (typically, the floor or medial wall is affected in this type of fracture), the inferior oblique and inferior rectus muscles often get caught in the fracture, limiting eye movements and causing double vision. Often, no operative procedure is necessary for healing because, as the inflammation subsides, the eye muscles become freed from the bone and normal eye movements resume.

Copyright © 2016 Wolters Kluwer

Vilensky_07.indd 370

3/17/15 1:20 AM

Head and Neck

7.35

1 5 2 6 3

7

4

Vilensky_07.indd 371

3/17/15 1:20 AM

I. LABELS 1. 2. 3. 4. 5. 6. 7.

7.35

Upper lip and orbicularis oris muscle Genioglossus muscle Geniohyoid muscle Anterior belly of digastric muscle Uvula Lingual tonsil Epiglottis

II. CLINICAL ANATOMY The mylohyoid and the anterior belly of the digastric muscles are both innervated by the mylohyoid nerve, which is a branch of the inferior alveolar nerve. The mylohyoid nerve branches off the inferior alveolar nerve before the latter enters the mandibular canal. Both the mylohyoid nerve and the inferior alveolar nerve are ultimately branches of the mandibular nerve, which is the third division of the trigeminal nerve. Thus, injury to the mandibular nerve will paralyze these muscles as well as the muscles of mastication.

Copyright © 2016 Wolters Kluwer

Vilensky_07.indd 372

3/17/15 1:20 AM

Head and Neck

7.36

1

6

2

7

3 4 5

Vilensky_07.indd 373

3/17/15 1:20 AM

I. LABELS 1. 2. 3. 4. 5. 6. 7.

7.36

Styloglossus muscle Lingual nerve (V3) Submandibular ganglion Hyoglossus muscle Hypoglossal nerve (n. XII) Buccinator muscle Tongue

II. CLINICAL ANATOMY During chewing, the buccinator muscles contract to keep food out of the recess between the teeth and the cheek. If the muscles are paralyzed due to a lesion of the facial nerve, the patient will use his or her fingers to remove food from this recess.

Copyright © 2016 Wolters Kluwer

Vilensky_07.indd 374

3/17/15 1:20 AM

Head and Neck

7.37

4 5 1 2

3

Vilensky_07.indd 375

3/17/15 1:20 AM

7.37

I. LABELS 1. 2. 3. 4. 5.

Submandibular duct Sublingual gland Geniohyoid muscle Lingual nerve Chorda tympani

II. QUESTION A patient with a severe middle ear infection has lost function of his chorda tympani nerve. Which of the following is most likely to result from this lesion? Decreased Decreased Decreased Decreased Decreased

parotid gland secretion submandibular gland secretion lacrimal gland secretion nasal glands secretion laryngeal glands secretion

Copyright © 2016 Wolters Kluwer

Vilensky_07.indd 376

Answer is B

A. B. C. D. E.

3/17/15 1:20 AM

Head and Neck

7.38

3 1

4

2 5

6

7

Vilensky_07.indd 377

3/17/15 1:20 AM

I. LABELS 1. 2. 3. 4. 5. 6. 7.

7.38

Accessory nerve Superior cervical ganglion of sympathetic trunk Stylopharyngeal muscle Superior constrictor muscle of pharynx Middle constrictor muscle of pharynx Inferior constrictor muscle of pharynx Esophagus

II. QUESTION A tumor in the apex of the left lung is compressing the patient’s cervical sympathetic trunk. The patient would likely have which of the following conditions on his left side? Papilledema Horner syndrome Bell palsy Dysphagia Dry eyes

Copyright © 2016 Wolters Kluwer

Vilensky_07.indd 378

Answer is B

A. B. C. D. E.

3/17/15 1:20 AM

Head and Neck

7.39

1

2

4

3 5

Vilensky_07.indd 379

3/17/15 1:20 AM

I. LABELS 1. 2. 3. 4. 5.

7.39

Hyoid bone Thyrohyoid muscle Cricothyroid muscle Thyroid cartilage Cricoid cartilage

II. CLINICAL ANATOMY During puberty, the thyroid cartilage markedly increases in size under the influence of testosterone, producing the “Adam’s apple” in men. Growth of the thyroid cartilage is associated with an increase in the length of the vocal chords, resulting in a lower frequency voice in men compared with young boys.

Copyright © 2016 Wolters Kluwer

Vilensky_07.indd 380

3/17/15 1:20 AM

Head and Neck

7.40

4 5 1 6 2

3

7

Vilensky_07.indd 381

3/17/15 1:20 AM

I. LABELS 1. 2. 3. 4. 5. 6. 7.

7.40

Cuneiform tubercle Superior laryngeal nerve Posterior cricoarytenoid muscle Epiglottis Aryepiglottic fold Aryepiglottic muscle Inferior laryngeal nerve (branch of recurrent nerve)

II. QUESTION While performing carotid endarterectomy, a surgeon immediately realizes that she inadvertently cut the patient’s left superior laryngeal nerve. Which of the following is most likely? A. The patient is more likely to aspirate food. B. The patient will have difficulty abducting his vocal folds. C. The patient will have difficulty adducting his vocal folds. D. The patient will lose taste from the posterior one-third of his tongue. E. The patient will be unable to elevate his larynx during swallowing.

Vilensky_07.indd 382

Answer is A

Copyright © 2016 Wolters Kluwer

3/17/15 1:20 AM

Head and Neck

7.41 5

1

2

6 7

8

9 3

4

Vilensky_07.indd 383

3/17/15 1:20 AM

I. LABELS

7.41

Facial artery Mylohyoid muscle Omohyoid muscle Sternohyoid muscle Stylohyoid muscle Posterior belly of digastric muscle Superior cervical lymph nodes Superior laryngeal artery and internal branch of superior laryngeal nerve 9. Internal jugular vein

1. 2. 3. 4. 5. 6. 7. 8.

II. QUESTION A tumor in the jugular foramen that is compressing the internal jugular vein might also be compressing which of the following groups of cranial nerves? VIII and IX IX, X, and XI VII and VIII VI and VII X, XI, and XII

Copyright © 2016 Wolters Kluwer

Vilensky_07.indd 384

Answer is B

A. B. C. D. E.

3/17/15 1:20 AM

Head and Neck

7.42

1 2

6 3

4

7

5

Vilensky_07.indd 385

3/17/15 1:20 AM

I. LABELS 1. 2. 3. 4. 5. 6. 7.

7.42

Anterior belly of digastric muscle Mylohyoid muscle Thyrohyoid muscle Sternothyroid muscle Thyroid gland Omohyoid muscle Sternohyoid muscle

II. CLINICAL ANATOMY The ansa cervicalis is composed of a series of loops of nerve twigs from C1 to C3 that innervate the infrahyoid muscles, sternohyoid, sternothyroid, and omohyoid. The superior root of the ansa cervicalis briefly travels with and is affixed to the hypoglossal nerve, but there is no intermingling of fibers between the two nerves. Thus, it is incorrect to suggest that any of the infrahyoid muscles are innervated by the hypoglossal nerve (cranial nerve XII), which does innervate almost all of the muscles of the tongue. Clearly then, a brainstem lesion that affects the hypoglossal nucleus would not affect the actions of these muscles.

Copyright © 2016 Wolters Kluwer

Vilensky_07.indd 386

3/17/15 1:20 AM

Head and Neck

7.43

1

2

3

5

4

6 7

Vilensky_07.indd 387

3/17/15 1:20 AM

I. LABELS 1. 2. 3. 4. 5. 6. 7.

7.43

Tongue Lateral glossoepiglottic fold Inferior constrictor muscle of pharynx Posterior cricoarytenoid muscle Internal branch of superior laryngeal nerve Sympathetic trunk Inferior laryngeal nerve

II. CLINICAL ANATOMY The inferior laryngeal nerve is the continuation of the recurrent laryngeal nerve inside the larynx. This nerve innervates all of the laryngeal muscles except the cricothyroid and also provides sensory innervation to the mucosa inferior to the ventricle.

Copyright © 2016 Wolters Kluwer

Vilensky_07.indd 388

3/17/15 1:20 AM

Brain 1

8.1 5

2 3

6

4

7

Vilensky_08.indd 389

3/17/15 12:34 AM

I. LABELS 1. 2. 3. 4. 5. 6. 7.

8.1

Falx cerebri Position of middle meningeal artery Optic nerve Internal carotid artery Superior sagittal sinus Straight sinus Cerebellar tentorium

II. CLINICAL ANATOMY The middle meningeal artery is located between the dura mater and the inner surface of the cranium. Its anterior branch crosses the pterion, which is a vulnerable area where the bones of the calvaria are very thin. Fracture of the calvaria at the pterion can tear the middle meningeal artery, resulting in an epidural hematoma, a life-threatening condition that requires emergency treatment to relieve excess intracranial pressure caused by the accumulation of blood in the epidural space.

Copyright © 2016 Wolters Kluwer

Vilensky_08.indd 390

3/17/15 12:34 AM

Brain

8.2 4

1 2

5

3

Vilensky_08.indd 391

3/17/15 12:34 AM

I. LABELS 1. 2. 3. 4. 5.

8.2

Precentral gyrus Central sulcus Postcentral gyrus Longitudinal fissure Arachnoid granulations

II. QUESTION Obstruction of the arachnoid granulations prevents cerebrospinal fluid from being reabsorbed into the dural venous sinuses, which would result in which of the following conditions? Cerebral edema Communicating (nonobstructive) hydrocephalus Noncommunicating (obstructive) hydrocephalus Meningitis Intracranial hemorrhage

Copyright © 2016 Wolters Kluwer

Vilensky_08.indd 392

Answer is B

A. B. C. D. E.

3/17/15 12:34 AM

Brain 1

2

8.3 7

8 9

3 4

10

5

6

Vilensky_08.indd 393

3/17/15 12:34 AM

I. LABELS 1. 2. 3. 4. 5. 6. 7. 8. 9. 10.

8.3

Frontal lobe Temporal lobe Mammillary body Pons Pyramid Cerebellum Olfactory bulb Optic nerve Optic tract Trigeminal nerve (n. V)

II. QUESTION The optic chiasm is located superior to the pituitary gland and may be compressed by pituitary adenomas. What pattern of vision loss would result from compression of the optic chiasm? Complete blindness in both eyes Binasal hemianopsia Bitemporal hemianopsia Superior hemianopsia in both eyes Inferior hemianopsia in both eyes

Copyright © 2016 Wolters Kluwer

Vilensky_08.indd 394

Answer is C B

A. B. C. D. E.

3/17/15 12:34 AM

Brain

8.4

1

2 5 3

6 7

4

Vilensky_08.indd 395

3/17/15 12:34 AM

I. LABELS 1. 2. 3. 4. 5. 6. 7.

8.4

Olfactory bulb and tract Optic nerve Pedunculus cerebri Corpus callosum Infundibulum Substantia nigra Parahippocampal gyrus

II. CLINICAL ANATOMY The substantia nigra is a collection of dopaminergic neurons located in the midbrain. It is part of the basal ganglia, a collection of subcortical nuclei that regulate motor function. Death of the dopaminergic neurons in the substantia nigra is one of the characteristics of Parkinson’s disease. Symptoms of Parkinson’s disease include tremor, akinesia, bradykinesia, stiffness, and masked facies.

Copyright © 2016 Wolters Kluwer

Vilensky_08.indd 396

3/17/15 12:34 AM

Brain

8.5

1

2

5

3

6

4 7

Vilensky_08.indd 397

3/17/15 12:34 AM

I. LABELS 1. 2. 3. 4. 5. 6. 7.

8.5

Precentral gyrus Central sulcus Lateral sulcus Temporal lobe Postcentral gyrus Occipital lobe Cerebellum

II. CLINICAL ANATOMY The precentral gyrus is the primary motor cortex, which controls volitional movements on the contralateral side of the body and face. The postcentral gyrus is the primary somatosensory cortex, which processes sensation from the contralateral side of the body and face. Both of these areas contain a topographical representation of the body (homunculus), with the face located laterally and the legs and feet located medially. Lesions to the pre- and/or postcentral gyrus result in a loss of motor control and/or sensation on the contralateral side of the face and/or body.

Copyright © 2016 Wolters Kluwer

Vilensky_08.indd 398

3/17/15 12:34 AM

Brain

8.6

1 5 2 6 3 7 4 8

Vilensky_08.indd 399

3/17/15 12:34 AM

I. LABELS 1. 2. 3. 4. 5. 6. 7. 8.

8.6

Thalamus Occipital lobe Cerebellum Medulla oblongata Corpus callosum Hypothalamus Pons Spinal cord

II. QUESTION A 45-year-old man has been experiencing disruptions in his sleep–wake cycle that were found to be a result of abnormal nocturnal surges in melatonin secretion. Which area of the brain is involved in regulating circadian rhythms? Hypothalamus Insular cortex Midbrain Pons Thalamus

Copyright © 2016 Wolters Kluwer

Vilensky_08.indd 400

Answer is A B

A. B. C. D. E.

3/17/15 12:34 AM

Brain

8.7 4

1 2

5

3

6

Vilensky_08.indd 401

3/17/15 12:34 AM

I. LABELS 1. 2. 3. 4. 5. 6.

8.7

Superior cerebellar peduncle Middle cerebellar peduncle Cerebellar tonsil Vermis Nodule of vermis Flocculus of cerebellum

II. QUESTION A 58-year-old homeless man was brought to the ER by some good Samaritans who found him collapsed in the street. He has a 35-year history of alcohol abuse, and on physical examination, he is unable to sit up without steadying himself (truncal ataxia) and he walks with a wide-based gait. He is diagnosed with a cerebellar ataxia brought on by chronic alcoholism. Which region of the cerebellum is likely affected in this patient? Cerebellar hemispheres Cerebellar tonsils Cerebellar vermis Flocculonodular lobe Middle cerebellar peduncles

Copyright © 2016 Wolters Kluwer

Vilensky_08.indd 402

Answer is C

A. B. C. D. E.

3/17/15 12:34 AM

Brain

8.8 4 5 6

1 2 3

Vilensky_08.indd 403

3/17/15 12:34 AM

I. LABELS 1. 2. 3. 4. 5. 6.

8.8

Nodule of vermis Middle cerebellar peduncle Flocculus of cerebellum Culmen of vermis Primary fissure of cerebellum Declive of vermis

II. CLINICAL ANATOMY The cerebellum plays an important role in motor coordination. Patients who have cerebellar disorders present with ataxia, which is a generalized syndrome of incoordination involving gait and limb movements, slurred speech, and oculomotor disturbances.

Copyright © 2016 Wolters Kluwer

Vilensky_08.indd 404

3/17/15 12:34 AM

Brain

8.9

1 2

5

6 3 7

4

8 9

Vilensky_08.indd 405

3/17/15 12:34 AM

I. LABELS 1. 2. 3. 4. 5. 6. 7. 8. 9.

8.9

Caudate nucleus Lentiform nucleus Pons Pyramid of medulla Olfactory tract Optic nerve (n. II) Trigeminal nerve (n. V) Inferior olive Decussation of pyramids

II. QUESTION A 63-year-old man was referred to a neurologist after suffering a stroke. He presents with hemiparesis and loss of proprioception, both on his left side, and when asked to protrude his tongue, it deviates to the right. What is the most likely location of the lesion? Left side of the medulla Left side of the pons Right side of the medulla Right side of the pons Right side of midbrain

Copyright © 2016 Wolters Kluwer

Vilensky_08.indd 406

Answer is C

A. B. C. D. E.

3/17/15 12:34 AM

Brain

8.10

1

2 4 5

3 6

Vilensky_08.indd 407

3/17/15 12:34 AM

I. LABELS 1. 2. 3. 4. 5. 6.

8.10

Thalamus Epiphysis (pineal gland) Fourth ventricle Superior colliculus Inferior colliculus Middle cerebellar peduncle

II. QUESTION A 40-year-old man presents with an upward gaze palsy. Attempts at upward gaze caused a convergence-retraction nystagmus. Accommodation reflex was present, but pupillary light reflex was absent. Which of the following best explains these symptoms? Dorsal midbrain (Parinaud) syndrome Lateral medullary (Wallenberg) syndrome Medial medullary syndrome Paramedian midbrain (Benedikt) syndrome Superior alternating hemiplegia (Weber syndrome)

Copyright © 2016 Wolters Kluwer

Vilensky_08.indd 408

Answer is A

A. B. C. D. E.

3/17/15 12:34 AM

Brain

1

8.11

5

2 6 3

7

4

Vilensky_08.indd 409

3/17/15 12:34 AM

I. LABELS 1. 2. 3. 4. 5. 6. 7.

8.11

Anterior cerebral artery Middle cerebral artery Posterior cerebral artery Vertebral artery Anterior communicating artery Internal carotid artery Basilar artery

II. CLINICAL ANATOMY The brain receives its blood supply from two sources: the internal carotid arteries provide the anterior circulation and the vertebral arteries provide the posterior circulation. Anterior and posterior communicating arteries connect the two circulations, forming the cerebral arterial circle (of Willis), which provides collateral circulation in the event that one of the arteries becomes blocked.

Copyright © 2016 Wolters Kluwer

Vilensky_08.indd 410

3/17/15 12:34 AM

Brain

8.12

1 2 3

5 6

4

Vilensky_08.indd 411

3/17/15 12:34 AM

I. LABELS 1. 2. 3. 4. 5. 6.

8.12

Septum pellucidum Anterior cerebral artery Posterior cerebral artery Basilar artery Cerebral aqueduct Fourth ventricle

II. QUESTION A stroke involving the right anterior cerebral artery would have the most significant effect on which region of the body? Left arm Right arm Left leg Right leg Left side of face

Copyright © 2016 Wolters Kluwer

Vilensky_08.indd 412

Answer is C

A. B. C. D. E.

3/17/15 12:34 AM

Brain

8.13

1

2 3

6

4

7

5

8

Vilensky_08.indd 413

3/17/15 12:34 AM

I. LABELS 1. 2. 3. 4. 5. 6. 7. 8.

8.13

Internal carotid artery Vertebral artery Right common carotid artery Right subclavian artery Brachiocephalic trunk Left common carotid artery Left subclavian artery Aortic arch

II. CLINICAL ANATOMY Carotid stenosis is a narrowing of the carotid arteries caused by the buildup of atherosclerotic plaques. The carotid bifurcation is a common location of atherosclerosis. From this location, emboli from an atherosclerotic plaque can travel to the brain and occlude cerebral circulation. A transient ischemic attack (TIA), sometimes called a mini-stroke, is often a symptom of carotid stenosis. A TIA differs from a stroke in that blood flow to the brain is only interrupted temporarily, and any neurological deficits resolve within 24 hours. TIAs should not be ignored, however, as they are often “warning signs” of a stroke.

Copyright © 2016 Wolters Kluwer

Vilensky_08.indd 414

3/17/15 12:34 AM

Brain

8.14

1 2

5

3 6 4

7

8

Vilensky_08.indd 415

3/17/15 12:34 AM

I. LABELS 1. 2. 3. 4. 5. 6. 7. 8.

8.14

Genu of corpus callosum Head of caudate nucleus Stria terminalis Choroid plexus of lateral ventricle Putamen Pes hippocampi Crus of fornix Splenium of corpus callosum

II. QUESTION A 35-year-old man visits his physician because he had been experiencing sporadic involuntary movements and was becoming increasingly uncoordinated. Genetic testing revealed that the man had Huntington’s disease, an autosomal dominant disease caused by a loss of GABA-ergic neurons in the basal ganglia. Which of the following structures is most affected by this disease? Caudate nucleus/putamen Globus pallidus Substantia nigra Subthalamic nucleus Thalamus

Copyright © 2016 Wolters Kluwer

Vilensky_08.indd 416

Answer is A

A. B. C. D. E.

3/17/15 12:34 AM

Brain

8.15

1 4 2 3

5 6

Vilensky_08.indd 417

3/17/15 12:34 AM

8.15

I. LABELS 1. 2. 3. 4. 5. 6.

Pulvinar of thalamus Mammillary body Anterior commissure Fornix Hippocampal fimbria Pes hippocampi

II. QUESTION While driving to work one morning, an ER nurse suddenly experienced an intense feeling of déjà vu, accompanied by the smell of burning rubber. The feeling passed after a few minutes and the rest of her commute was uneventful. When she arrived at the hospital, she mentioned the experience to one of the physicians who suggested that she may have had a seizure. Where is the most likely location of the seizure? Frontal lobe Parietal lobe Occipital lobe Temporal lobe Generalized seizure with no specific location

Copyright © 2016 Wolters Kluwer

Vilensky_08.indd 418

Answer is D

A. B. C. D. E.

3/17/15 12:34 AM

Brain

8.16

1 2 5 3 6 4

Vilensky_08.indd 419

7

3/17/15 12:34 AM

I. LABELS 1. 2. 3. 4. 5. 6. 7.

8.16 8.15

Anterior horn of lateral ventricle Interventricular foramen of Monro Third ventricle Inferior horn of lateral ventricle Posterior horn of lateral ventricle Cerebral aqueduct Fourth ventricle

II. CLINICAL ANATOMY Hydrocephalus is a condition in which there is excessive accumulation of cerebrospinal fluid (CSF) in the brain. There are two main types of hydrocephalus: communicating and noncommunicating. Communicating hydrocephalus occurs when the flow of CSF is blocked after it exits the ventricles, hence communication between the ventricles is intact. Noncommunicating or “obstructive” hydrocephalus occurs when the flow of CSF is blocked along one of the passages that connects the ventricles. Hydrocephalus is most often treated by surgically inserting a shunt to drain excess CSF from the brain.

Copyright © 2016 Wolters Kluwer

Vilensky_08.indd 420

3/17/15 12:34 AM

Brain

8.17

1 4 2

5

3 6

Vilensky_08.indd 421

3/17/15 12:34 AM

I. LABELS 1. 2. 3. 4. 5. 6.

8.17

Corpus callosum Internal capsule Anterior commissure Head of caudate nucleus Column of fornix Amygdaloid body

II. QUESTION Bilateral lesions of the anterior portion of the temporal lobes would most likely result in which of the following? Broca’s aphasia Hemineglect syndrome Klüver-Bucy syndrome Prosopagnosia Wernicke’s aphasia

Copyright © 2016 Wolters Kluwer

Vilensky_08.indd 422

Answer is C

A. B. C. D. E.

3/17/15 12:34 AM

Brain

8.18

4 1 2 3

Vilensky_08.indd 423

5

6

3/17/15 12:34 AM

I. LABELS 1. 2. 3. 4. 5. 6.

8.18

Putamen Globus pallidus Amygdala Caudate nucleus Thalamus Third ventricle

II. CLINICAL ANATOMY The thalamus is a relay station for information traveling to and from the cerebral cortex. All sensory information, except olfaction, is relayed through the thalamus before it reaches the cortex. Thalamic strokes often result in thalamic pain syndrome (Dejerine-Roussy syndrome) and cause a contralateral loss of sensation, accompanied by pain, which is often constant and can range from mild to severe in intensity.

Copyright © 2016 Wolters Kluwer

Vilensky_08.indd 424

3/17/15 12:34 AM

Brain

1 2

8.19

3

4 5

Vilensky_08.indd 425

3/17/15 12:34 AM

I. LABELS 1. 2. 3. 4. 5.

8.19

Corpus callosum Choroid plexus of lateral ventricle Lateral ventricle Cerebral aqueduct Pons and transverse pontine fibers

II. QUESTION Locked-in syndrome could be caused by a stroke involving which of the following vessels? Anterior cerebral artery Middle cerebral artery Basilar artery Lenticulostriate arteries Posterior inferior cerebellar artery

Copyright © 2016 Wolters Kluwer

Vilensky_08.indd 426

Answer is C B

A. B. C. D. E.

3/17/15 12:34 AM

Brain

8.20

1 6 2 3 7

4

8

5

Vilensky_08.indd 427

3/17/15 12:34 AM

I. LABELS 1. 2. 3. 4. 5. 6. 7. 8.

8.20

Fourth ventricle Hypoglossal nucleus Medial longitudinal fasciculus Inferior olivary nucleus Corticospinal fibers Vestibular nuclei Reticular formation Medial lemniscus

II. QUESTION A patient presents with loss of pain and temperature sensation on the right side of his body and on the left side of his face. He is also experiencing difficulty swallowing (dysphagia), slurred speech (dysarthria), and vertigo. A lesion of which vessel would cause these symptoms? Left anterior spinal artery Left anterior inferior cerebellar artery Right anterior inferior cerebellar artery Left posterior inferior cerebellar artery Right posterior inferior cerebellar artery

Copyright © 2016 Wolters Kluwer

Vilensky_08.indd 428

Answer is D

A. B. C. D. E.

3/17/15 12:34 AM

Brain

8.21

1 2

5

3 4

6

7

Vilensky_08.indd 429

3/17/15 12:34 AM

I. LABELS 1. 2. 3. 4. 5. 6. 7.

8.21

Head of caudate nucleus Putamen Globus pallidus Thalamus Anterior limb of internal capsule Posterior limb of internal capsule Posterior horn of lateral ventricle

II. QUESTION A patient presents with flailing, ballistic-type movements of his limbs that originate from the shoulder and hip. A lesion of which of the following structures could cause this type of movement disorder? Caudate nucleus Putamen Substantia nigra Subthalamic nucleus Thalamus

Copyright © 2016 Wolters Kluwer

Vilensky_08.indd 430

Answer is D

A. B. C. D. E.

3/17/15 12:34 AM

Brain

1

8.22

4

2 5 3 6

Vilensky_08.indd 431

3/17/15 12:34 AM

I. LABELS 1. 2. 3. 4. 5. 6.

8.22

Head of caudate nucleus Putamen Thalamus Genu of corpus callosum Internal capsule Splenium of corpus callosum

II. QUESTION A 74-year-old woman recently suffered a mild stroke. Upon returning home from the hospital and resuming her normal activities, her husband noticed some unusual behavior. She only combed her hair on the right side of her head and only applied makeup to the right side of her face. She also only ate from the right side of her plate and seemed to completely ignore her husband when he addressed her from her left. Where did her stroke most likely occur to cause these behaviors? Corpus callosum Right cerebellum Right precentral gyrus Right postcentral gyrus Right posterior parietal cortex

Copyright © 2016 Wolters Kluwer

Vilensky_08.indd 432

Answer is E

A. B. C. D. E.

3/17/15 12:34 AM

Brain

8.23

1

2 5

3 6

4 7

Vilensky_08.indd 433

3/17/15 12:34 AM

I. LABELS 1. 2. 3. 4. 5. 6. 7.

8.23

Optic nerve Hypophysis (pituitary gland) Cerebral aqueduct (beginning of fourth ventricle) Straight sinus Oculomotor nerve Vermis of cerebellum Transverse sinus

II. CLINICAL ANATOMY Papilledema is a condition in which the optic disc swells as a result of increased intracranial pressure. The optic nerve sheath is continuous with the subarachnoid space of the brain, hence increases in intracranial pressure can be transmitted to the optic disc. Papilledema causes engorgement of the retinal veins and hemorrhages around the optic disc. Prolonged papilledema can cause blindness if the underlying cause is not addressed.

Copyright © 2016 Wolters Kluwer

Vilensky_08.indd 434

3/17/15 12:34 AM

Brain

8.24

1

2

3

4 5

6

Vilensky_08.indd 435

3/17/15 12:34 AM

I. LABELS 1. 2. 3. 4. 5. 6.

8.24

Optic nerve Hypophysis (pituitary gland) Pons Temporal lobe Cerebellar tentorium Vermis of cerebellum

II. CLINICAL ANATOMY The cerebellar tentorium is a fold of dura that separates the cerebrum, which is located in the supratentorial space, from the cerebellum, which is located in the infratentorial space. There is an opening, the tentorial incisure, which allows for passage of the brainstem. Increased intracranial pressure in the supratentorial space can cause parts of the temporal lobe, particularly the uncus, to herniate through the tentorial incisure (uncal herniation). The herniated uncus puts pressure on brainstem structures, causing pupillary dilation, ophthalmoplegia, and hemiparesis.

Copyright © 2016 Wolters Kluwer

Vilensky_08.indd 436

3/17/15 12:34 AM

Brain

8.25

1 6 2 7 3

8

4 9 5

Vilensky_08.indd 437

3/17/15 12:34 AM

I. LABELS 1. 2. 3. 4. 5. 6. 7. 8. 9.

8.25

Corpus callosum Thalamus Cerebellum Fourth ventricle Spinal cord Anterior cerebral artery Hypothalamus Pons Medulla oblongata

II. QUESTION A left posterior cerebral artery stroke involving the left primary visual cortex would result in which pattern of vision loss? Blindness in the left eye Blindness in the right eye Left homonymous hemianopsia Right homonymous hemianopsia Right homonymous hemianopsia with macular sparing

Copyright © 2016 Wolters Kluwer

Vilensky_08.indd 438

Answer is E

A. B. C. D. E.

3/17/15 12:34 AM

Brain

8.26

1 6 7 2 3 4

8 9

5

Vilensky_08.indd 439

3/17/15 12:34 AM

I. LABELS 1. 2. 3. 4. 5. 6. 7. 8. 9.

8.26

Parietal lobe Occipital lobe Cerebellum Fourth ventricle Spinal cord Frontal lobe Lateral ventricle Hypophysial fossa with pituitary gland Sphenoid sinus

II. CLINICAL ANATOMY Because the hypophysial fossa lies immediately posterior to the sphenoid sinus, transsphenoidal endoscopic surgery is a minimally invasive technique for accessing the pituitary gland to remove tumors. An endoscope is inserted through the nose, and small openings are made through the walls of the sphenoid sinus and hypophysial fossa to reach the pituitary gland.

Copyright © 2016 Wolters Kluwer

Vilensky_08.indd 440

3/17/15 12:34 AM